Крок 1 - Медицина 2011 (буклет)

1 / 200
До хірургічного відділення доставлено чоловіка 35-ти років з гнійною раною на шиї попереду трахеї (в ділянці передвісцерального простору). Куди може розповсюдитись інфекція, якщо хворому терміново не зроблять операцію? A 35-year-old man was brought to the surgical department with a purulent wound on the neck in front of the trachea (in the area of the previsceral space). Where can the infection spread if the patient is not treated urgently operation?

В ретровісцеральний простір Into retrovisceral space

В грудну порожнину - переднє середостіння Into the chest cavity - anterior mediastinum

В надгрудинний міжапоневротичний простір In the suprasternal interaponeurotic space

В грудну порожнину - заднє середостіння Into the chest cavity - posterior mediastinum

В грудну порожнину - середнє середостіння Into the chest cavity - middle mediastinum

2 / 200
Внаслідок інсульту (крововилив в головний мозок) у хворого відсутні вольові рухи м’язів голови і шиї. Обстеження головного мозку за допомогою ЯМР показало, що гематома знаходиться в коліні внутрішньої капсули. Який провідний шлях пошкоджено у хворого? As a result of a stroke (hemorrhage in the brain), the patient has no voluntary movements of the muscles of the head and neck. An MRI examination of the brain showed that the hematoma is in the knee of the internal capsule. Which conducting path is damaged in the patient?

Tr.cortico-nuclearis Tr.cortico-nuclearis

Tr.thalamo-corticalis Tr.thalamo-corticalis

Tr.cortico-thalamicus Tr.cortico-thalamicus

Tr.cortico-spinalis Tr.cortico-spinalis

Tr.cortico-fronto-pontinus Tr.cortico-fronto-pontinus

3 / 200
Хворому з гіперсекрецією шлункового соку лікар рекомендував виключити з дієти насичені бульйони і овочеві відвари, тому що вони стимулюють шлункову секрецію переважно через активацію: To a patient with hypersecretion of gastric juice, the doctor recommended excluding saturated broths and vegetable broths from the diet, because they stimulate gastric secretion mainly through activation:

Смакових рецепторів Taste buds

Вироблення секретину Production of secretin

Механорецепторів ротової порожнини Oral cavity mechanoreceptors

Механорецепторів шлунка Gastric mechanoreceptors

Вироблення гастрину Production of gastrin

4 / 200
У хворого після короткочасного оперативного втручання, проведеного із застосуванням дитиліну, понад 30 хвилин відмічалось пригнічення дихання, не відновився попередній тонус м’язів. Яку допомогу необхідно надати хворому? After a short-term surgical intervention performed with the use of dithyline, the patient had respiratory depression for more than 30 minutes, and the previous muscle tone did not recover. What help should be given to the patient?'

Переливання крові Blood transfusion

Форсований діурез Forced diuresis

Гемосорбція Hemosorption

Перитонеальний діаліз Peritoneal dialysis

Гемодіаліз Hemodialysis

5 / 200
Хворий з діагнозом 'вогнищевий туберкульоз верхньої долі правої легені' в складі комбінованої терапії одержує ізоніазид. Через деякий час пацієнт почав пред’являти скарги на м’язову слабкість, зниження шкірної чутливості, порушення зору, координації рухів. Який вітамінний препарат доцільно використати для усунення даних явищ? A patient with a diagnosis of 'focal tuberculosis of the upper lobe of the right lung' receives isoniazid as part of combined therapy. After some time, the patient began to complain of muscle weakness, decreased skin sensitivity, impaired vision, coordination of movements. What vitamin preparation should be used to eliminate these phenomena?

Вітамін A Vitamin A

Вітамін D Vitamin D

Вітамін В6 Vitamin B6

Вітамін В12 Vitamin B12

Вітамін C Vitamin C

6 / 200
Хворому на гіпертонічну хворобу з супутнім обструктивним бронхітом призначили гіпотензивний засіб. Через деякий час у пацієнта почали з’являтися напади ядухи, розвинулась виражена брадикардія. На ЕКГ відмічались ознаки порушення атріовентрикулярного проведення. При призначенні якого препарату найбільш імовірний розвиток подібних симптомів? A hypertensive patient with concomitant obstructive bronchitis was prescribed a hypotensive agent. After some time, the patient began to have seizures, severe bradycardia developed. The ECG showed signs of a violation atrioventricular conduction. When prescribing which drug, the development of such symptoms is most likely?

Клофелін Clofelin

Верапаміл Verapamil

Анаприлін Anaprilin

Резерпін Reserpin

Корданум Cordanum

7 / 200
Хворий 60-ти років впродовж 9-ти років хворіє на цукровий діабет, отримує для корекції гіперглікемії інсулін-семіленте. 10 днів тому почав лікування гіпертонічної хвороби анаприліном. Через годину після прийому антигіпертензивного препарату розвинулась гіпоглікемічна кома. Який механізм виникнення гіпоглікемії за умови прийому анаприліну? A 60-year-old patient has been suffering from diabetes for 9 years, receives insulin-semilent to correct hyperglycemia. 10 days ago, he started treating hypertension with anaprilin. an hour after taking an antihypertensive drug, a hypoglycemic coma developed. What is the mechanism of hypoglycemia when taking anaprilin?

Збільшення біодоступності інсуліну-семіленте Increasing the bioavailability of insulin-semilente

Зменшення періоду напіввиведення глюкагону Decreased glucagon half-life

Збільшення періоду напіввиведення інсуліну-семіленте Increase in half-life of insulin semilente

Пригнічення глікогенолізу Suppression of glycogenolysis

Зменшення всмоктування глюкози Decreased glucose absorption

8 / 200
Хворий надійшов до хірургічного відділення з діагнозом: гострий панкреатит. Розпочато консервативне лікування. Призначення якого препарату є патогенетично обґрунтованим? The patient was admitted to the surgical department with a diagnosis of acute pancreatitis. Conservative treatment was started. The prescription of which drug is pathogenetically justified?

Фібринолізин Fibrinolysin

Панкреатин Pancreatin

Контрикал Contrical

Трипсин Trypsin

Хімотрипсин Chymotrypsin

9 / 200
До приймального відділення доставлений хворий зі скаргами на сухість в роті, світлобоязнь та порушення зору. Об’єктивно: шкіра гіперемована, суха, зіниці розширені, тахікардія. При подальшому обстеженні був встановлений діагноз: отруєння алкалоїдами беладонни. Який лікарський засіб доцільно застосувати? A patient was brought to the reception department with complaints of dry mouth, photophobia, and impaired vision. Objectively: the skin is hyperemic, dry, pupils are dilated, tachycardia. At further during the examination, a diagnosis was established: poisoning by belladonna alkaloids. What medicine should be used?

Дипіроксим Dipiroxime

Армін Armin

Прозерин Proserin

Пілокарпін Pilocarpine

Ацеклідин Aceclidine

10 / 200
При обстеженні 2-місячного хлопчика педіатр звернув увагу, що плач дитини схожий на нявкання кішки, відзначаються мікроцефалія і вада серця. За допомогою цитогенетичного методу був встановлений каріотип - 46 XY, 5р- . На якій стадії мітозу досліджували каріотип хворого? During the examination of a 2-month-old boy, the pediatrician noticed that the child's cry was similar to the meowing of a cat, microcephaly and a heart defect were noted. Using the cytogenetic method, the karyotype was established - 46 XY, 5p- . At what stage of mitosis was the patient's karyotype studied?

Анафаза Anaphase

Прометафаза Prometaphase

Метафаза Metaphase

Телофаза Telophase

Профаза Prophase

11 / 200
Чоловік, що страждає на спадкову хворобу, одружився із здоровою жінкою. У них було 5 дітей, три дівчинки і два хлопчика. Усі дівчатка успадкували хворобу батька. Який тип спадкування цього захворювання? A man suffering from a hereditary disease married a healthy woman. They had 5 children, three girls and two boys. All the girls inherited the disease from their father. What type inheritance of this disease?

Аутосомно-домінантний Autosomal dominant

Домінантний, зчеплений з X -хромосомою Dominant, X-linked

Рецесивний, зчеплений з X -хромосомою X-linked recessive

Зчеплений з Y-хромосомою Y-linked

Аутосомно-рецесивний Autosomal recessive

12 / 200
Похідні птерину (аміноптерин і метотрексат) - є конкурентними інгібіторами дигідрофолатредуктази, внаслідок чого вони пригнічують регенерацію тетрагідрофолієвої кислоти з дигідрофолату. Ці лікарські засоби призводять до гальмування міжмолекулярного транспорту одновуглецевих груп. Біосинтез якого полімеру при цьому пригнічується? Pterin derivatives (aminopterin and methotrexate) are competitive inhibitors of dihydrofolate reductase, as a result of which they inhibit the regeneration of tetrahydrofolic acid from dihydrofolate. These drugs lead to inhibition of the intermolecular transport of one-carbon groups The biosynthesis of which polymer is suppressed?

Гомополісахариди Homopolysaccharides

ДНК DNA

Гангліозиди Gangliosides

Білок Protein

Глікозаміноглікани Glycosaminoglycans

13 / 200
До лікарні надійшов хворий із скаргами на головний біль, біль у м’язах під час руху, слабкість, температуру, набряк повік і обличчя. Лікар пов’язує цей стан із вживанням свинини, купленої у приватних осіб. Який попередній діагноз може поставити лікар? A patient came to the hospital with complaints of headache, muscle pain during movement, weakness, temperature, swelling of the eyelids and face. The doctor associates this the condition with the consumption of pork bought from private individuals. What preliminary diagnosis can a doctor make?

Теніоз Taeniosis

Опісторхоз Opistorchosis

Трихінельоз Trichinellosis

Теніарінхоз Taeniarhynchosis

Фасціольоз Fasciolosis

14 / 200
В шкірі виявлена щільна, рухома, чітко відмежована від оточуючих тканин пухлина. На розрізі вона білого кольору, представлена волокнистою тканиною. Мікроскопічно: хаотично переплетені колагенові волокна, клітин мало. Що це за пухлина? A dense, mobile tumor, clearly separated from the surrounding tissues, was detected in the skin. On cross-section, it is white in color, represented by fibrous tissue. Microscopically: chaotically intertwined collagen fibers, few cells . What kind of tumor is this?

Міома Myoma

Десмоїд Desmoid

Гістіоцитома Histiocytoma

Фіброма Fibroma

Дерматофіброма Dermatofibroma

15 / 200
На розтині тіла померлої виявлено морфологічні зміни: стеноз лівого атріовентрикулярного отвору, недостатність мітрального клапана. Гістологічно в міокарді - вогнищевий кардіосклероз, наявність квітучих гранульом Ашофф-Талалаєва. Який з перерахованих нижче діагнозів найбільш імовірний? The autopsy of the deceased revealed morphological changes: stenosis of the left atrioventricular opening, insufficiency of the mitral valve. Histologically, in the myocardium - focal cardiosclerosis, the presence of flowering Ashoff-Talalayev granulomas. Which of the of the diagnoses listed below, which is the most likely?

Системний червоний вовчак Systemic lupus erythematosus

Вузликовий періартеріїт Nodular periarteritis

Дерматоміозит Dermatomyositis

Склеродермія Scleroderma

Ревматизм Rheumatism

16 / 200
У постраждалого виявлено рану верхньої частини передньої ділянки плеча. При обстеженні встановлена втрата активного згинання в ліктьовому суглобі і чутливості шкіри передньобічної поверхні передпліччя. Порушення функції якого нерва має місце? The victim was found to have a wound in the upper part of the front part of the shoulder. During the examination, the loss of active flexion in the elbow joint and the sensitivity of the skin of the anterolateral surface of the forearm was detected. What nerve function is impaired?'

М’язово-шкірний Musculocutaneous

Серединний Middle

Пахвовий Axillary

Променевий Radial

Ліктьовий Cubit

17 / 200
При мікроскопічному дослідженні шийного лімфатичного вузла виявлено скупчення епітеліоїдних клітин, лімфоцитів і гігантських клітин Пирогова-Лангханса. В центрі - казеозний некроз. Вкажіть найбільш імовірну патологію: Microscopic examination of the cervical lymph node revealed a cluster of epithelioid cells, lymphocytes and Pirogov-Langhans giant cells. Caseous necrosis in the center. Specify the most likely pathology:

Риносклерома Rhinoscleroma

Сап Sap

Саркоїдоз Sarcoid

Туберкульоз Tuberculosis

Сифіліс Syphilis

18 / 200
При гістологічному дослідженні новоутворення шкіри виявлено: паренхіма сформована з покривного епітелію із збільшеним числом шарів. Строма разом з розростаннями епітелію формує сосочки. Вкажіть вид атипізму: During the histological examination of the skin neoplasm, it was found: the parenchyma is formed from the covering epithelium with an increased number of layers. The stroma together with the growths of the epithelium forms papillae. Specify the type of atypism:

Метаболічний Metabolic

Функціональний Functional

Тканинний Fabric

Гістохімічний Histochemical

Клітинний Cellular

19 / 200
Після перенесеного сепсису у хворої 27-ми років з’явився бронзовий колір шкіри, характерний для аддісонової хвороби. Механізм гіперпігментації полягає в підвищенні секреції такого гормону: After suffering sepsis, a 27-year-old patient developed a bronze skin color, characteristic of Addison's disease. The mechanism of hyperpigmentation is increased secretion of such a hormone:

Соматотропний Somatotropic

Тиреотропний Thyrotropic

Меланоцитстимулюючий Melanocyte stimulating

B-ліпотропний B-lipotropic

Гонадотропний Gonadotropic

20 / 200
У пацієнта, який півтора місяця тому переніс інфаркт міокарда, діагностовано синдром Дреслера з характерною тріадою: перикардит, плеврит, пневмонія. Який головний механізм цього ускладнення? A patient who suffered a myocardial infarction a month and a half ago was diagnosed with Dressler's syndrome with a characteristic triad: pericarditis, pleurisy, pneumonia. What is the main mechanism of this complication?

Викидання у кров міокардіальних ферментів Release of myocardial enzymes into the blood

Інтоксикація організму продуктами некрозу Intoxication of the body by products of necrosis

Активація сапрофітної мікрофлори Activation of saprophytic microflora

Зниження резистентності до інфекційних агентів Decreasing resistance to infectious agents

Сенсибілізація організму антигенами міокарда Sensitization of the body by myocardial antigens

21 / 200
Жінка після перенесеного нервового потрясіння погано спить. Якому снодійному засобу слід віддати перевагу для лікування безсоння? After a nervous breakdown, a woman does not sleep well. Which sleeping pill should be preferred for the treatment of insomnia?

Етамінал-натрій Ethaminal sodium

Фенобарбітал Phenobarbital

Барбітал Barbital

Нітразепам Nitrazepam

Барбаміл Barbamil

22 / 200
У хворої на дифтерію дитини через 10 днів після введення антитоксичної протидифтерійної сироватки з’явилася висипка на шкірі, яка супроводжувалася сильним свербежем, підвищенням температура тіла до 38oC, появою болю у суглобах. Яку причину цих явищ можна припустити? A child with diphtheria developed a rash on the skin 10 days after the administration of anti-toxic anti-diphtheria serum, which was accompanied by severe itching, an increase in body temperature up to 38oC, the onset of pain in the joints. What reason can be assumed for these phenomena?

Атопія Atopy

Сироваткова хвороба Serum sickness

Анафілактична реакція Anaphylactic reaction

Гіперчутливість уповільненого типу Delayed hypersensitivity

Контактна алергія Contact allergy

23 / 200
У хворого з розладом мозкового кровообігу порушений акт ковтання, він може поперхнутися при прийомі рідкої їжі. Який відділ мозку уражений? A patient with a disorder of cerebral blood circulation has a disturbed act of swallowing, he may gag when taking liquid food. Which part of the brain is affected?

Шийний відділ спинного мозку Cervical spinal cord

Проміжний мозок Midbrain

Довгастий мозок Olongate brain

Мозочок Cerebellum

Середній мозок Average brain

24 / 200
Молодий чоловік 19-ти років страждав з раннього дитинства на бронхоектатичну хворобу, помер від ниркової недостатності. На розтині окрім множинних бронхоектатичних каверн, заповнених гнійним ексудатом, виявлено збільшені в розмірах нирки щільної консистенції, кірковий шар потовщений, білого кольору, щільний. Піраміди нирки анемічні, чіткі. Назвіть процес, який розвинувся в нирках: A 19-year-old young man suffered from bronchiectatic disease since early childhood and died of kidney failure. At autopsy, in addition to multiple bronchiectatic cavities filled with purulent exudate, enlarged the size of a kidney with a dense consistency, the cortical layer is thickened, white, dense. The pyramids of the kidneys are anemic, clear. Name the process that developed in the kidneys:

Вторинний амілоїдоз Secondary amyloidosis

Гломерулонефрит Glomerulonephritis

Вроджений кістоз нирок Congenital renal cyst

Вторинний нефросклероз Secondary nephrosclerosis

Хронічний пієлонефрит Chronic pyelonephritis

25 / 200
Дитина 3-х років з множинними порушеннями розвитку кісток лицевого відділу черепа померла. Причина смерті - сепсис, який розвинувся на фоні бронхопневмонії. В крові вміст імуноглобулінів в межах фізіологічної норми. На розтині встановлена відсутність тимусу. Назвіть головну причину хвороби дитини: A 3-year-old child with multiple disorders of the development of the facial bones of the skull died. The cause of death was sepsis, which developed against the background of bronchopneumonia. The content of immunoglobulins in the blood was within the physiological range norms. An autopsy revealed the absence of a thymus. Name the main cause of the child's illness:

Синдром недостатності клітинного імунітету Cellular immune deficiency syndrome

Гострий лімфолейкоз Acute lymphocytic leukemia

Синдром хронічної інтоксикації Syndrome of chronic intoxication

Вторинний імунодефіцитний синдром Secondary immunodeficiency syndrome

Синдром комбінованого імунодефіциту Combined immunodeficiency syndrome

26 / 200
До лікарні доставлено пораненого вогнепальною зброєю з сильною кровотечею. При огляді хірургом установлено, що кульовий канал пройшов через передню стінку живота, склепіння шлунка i вийшов на рівні ІХ ребра по лівій середній пахвовій лінії. Який орган постраждав разом з пораненням шлунка? A gunshot wound with heavy bleeding was brought to the hospital. During the examination by the surgeon, it was established that the bullet channel passed through the front wall of the abdomen, the vault of the stomach and exited at the level of the 10th rib in the left middle axillary line. What organ was injured along with the stomach wound?

Селезінка Spleen

Поперечна ободова кишка Transverse colon

Підшлункова залоза Pancreas

Ліва частка печінки Left liver lobe

Ліва нирка Left kidney

27 / 200
У хворого із захворюванням печінки виявлено зниження вмісту протромбіну в крові. Це призведе, перш за все, до порушення: In a patient with liver disease, a decrease in the content of prothrombin in the blood was detected. This will lead, first of all, to a violation:

Другої фази коагуляційного гемостазу Second phase of coagulation hemostasis

Фібринолізу Fibrinolysis

Антикоагулянтних властивостей крові Anticoagulant properties of blood

Судинно-тромбоцитарного гемостазу Vascular-platelet hemostasis

Першої фази коагуляційного гемостазу First phase of coagulation hemostasis

28 / 200
в крові: ер.- 3,0 • 1012 28. /л; Hb- 90г/л; ретикул.- 0,5%. В мазку: пойкілоцити, гіпохромні еритроцити. Залізо сироватки крові - 80 мкмоль/л. Для якої патології це характерно? in the blood: er.- 3.0 • 1012 28. /l; Hb- 90g/l; reticul.- 0.5%. In the smear: poikilocytes, hypochromic erythrocytes. Blood serum iron - 80 μmol/l. What pathology is this typical for?

Хвороба Мінковського-Шоффара Minkowski-Shofar disease

Залізодефіцитна анемія Iron deficiency anemia

Серпоподібноклітинна анемія Sickle cell anemia

Б12-дефіцитна анемія B12 deficiency anemia

Залізорефрактерна анемія Iron refractory anemia

29 / 200
У фізично здорових молодих курсантів після важкого фізичного навантаження при одноденному пішому переході на 50 км в сечі виявлено білок, рівень якого в середньому не перевищував 1 г/л. Який різновид протеїнурії мав місце? In physically healthy young cadets, after heavy physical exertion during a one-day walking distance of 50 km, protein was detected in the urine, the level of which did not exceed 1 g/l on average. What kind of proteinuria took place?

Маршова Marshova

Аліментарна Alimony

Несправжня False

Дегідратаційна Dehydrating

Органічна Organic

30 / 200
Яким буде скорочення м’язів верхньої кінцівки при намаганні підняти непосильний вантаж? What will be the contraction of the muscles of the upper limb when trying to lift an overwhelming load?

Ізотонічне Isotonic

Фазичне Phase

Ауксотонічне Auxotonic

Ізометричне Isometric

Одиночне Single

31 / 200
У хворих на поворотний тиф виникає лихоманка, яка характеризується кількаденними періодами високої гарячки, що чергується з періодами нормальної температури. Така температурна крива називається: Patients with relapsing typhus develop fever, which is characterized by several-day periods of high fever alternating with periods of normal temperature. Such a temperature curve is called:

Febris intermittens Febris intermittents

Febris recurrens Febris recurrentens

Febris hectica Febris hectica

Febris atypica Febris atypica

Febris continua Febris continua

32 / 200
У хворого з алкогольним цирозом печінки скарги на загальну слабкість, задишку. Встановлено зниження артеріального тиску, асцит, розширення поверхневих вен передньої стінки живота, спленомегалію. Яке порушення гемодинаміки спостерігається у хворого? A patient with alcoholic cirrhosis of the liver complains of general weakness, shortness of breath. Decreased blood pressure, ascites, dilation of the superficial veins of the anterior abdominal wall, splenomegaly have been detected. What hemodynamic disorder is observed in the patient?

Недостатність лівого шлуночка серця Left ventricular failure

Недостатність правого шлуночка серця Right ventricular failure

Синдром портальної гіпертензії Portal hypertension syndrome

Колапс Collapse

Тотальна серцева недостатність Total heart failure

33 / 200
Під час обертання на каруселі у жінки 25-ти років з’явилися нудота, блювання, посилення потовиділення. Активація яких рецепторів зумовила рефлекторний розвиток цих симптомів? While rotating on a carousel, a 25-year-old woman developed nausea, vomiting, increased sweating. Activation of which receptors caused the reflex development of these symptoms?

Вестибулярні півколові канали Vestibular semicircular canals

Зорові Visual

Пропріорецептори скелетних м’язів Skeletal muscle proprioceptors

Кортієвого органу Corti's organ

Отолітові вестибуляри Otolith vestibules

34 / 200
У студента 18-ти років під час фізичного навантаження реографічно зареєстровано перерозподіл кровотоку органів. У яких судинах кровотік підвищився найбільшою мірою? In an 18-year-old student, redistribution of blood flow in organs was registered during physical activity. In which vessels did the blood flow increase the most?

Шлунково-кишковий тракт Gastrointestinal tract

Скелетні м’язи Skeletal muscles

Головний мозок Brain

Нирки Kidneys

Печінка Liver

35 / 200
Під час ревізії порожнини очеревини з приводу перитоніту виявлений обмежений гнійник у кореня брижі сигмоподібної кишки. У якому утворенні очеревини знаходиться гнійник? During the revision of the peritoneal cavity for peritonitis, a limited abscess was found at the root of the mesentery of the sigmoid colon. In which peritoneal formation is the abscess?

Лівий бічний канал Left Side Channel

Міжсигмоподібна ямка Intersigmoid fossa

Правий брижовий синус Right mesenteric sinus

Лівий брижовий синус Left mesenteric sinus

Правий бічний канал Right Side Channel

36 / 200
Під час розтину трупа шахтаря, що проробив у шахті більше 10-ти років, у легені виявлені тяжі білуватої волокнистої тканини і вузлики 0,2-0,3 см у діаметрі. При гістологічному дослідженні у вузликах невелика кількість коричневого пилу, концентричні розростання сполучної тканини, бідної на клітини, з вираженим гіалінозом. Про який пневмоконіоз слід думати у даному випадку? During the autopsy of the corpse of a miner who had worked in the mine for more than 10 years, masses of whitish fibrous tissue and nodules of 0.2-0.3 cm were found in the lungs in diameter. During histological examination, a small amount of brown dust in the nodules, concentric growths of connective tissue, poor in cells, with pronounced hyalinosis. What pneumoconiosis should be considered in this case?

Асбестоз Asbestosis

Сидероз Siderosis

Силікоз Silicosis

Талькоз Talkoz

Бериліоз Berylliosis

37 / 200
При ендоскопічному дослідженні сечового міхура проведена біопсія пухлини, що складається з тонких, розгалужених сосочків, вкритих декількома рядами клітин перехідного епітелію. Назвіть пухлину: During an endoscopic examination of the bladder, a biopsy was performed of a tumor consisting of thin, branched papillae covered by several rows of transitional epithelial cells. Name the tumor:

Перехідноклітинна карцинома Transitional cell carcinoma

Фіброаденома Fibroadenoma

Плоскоклітинна карцинома Squamous cell carcinoma

Базаліома Basal tumor

Папілома Papilloma

38 / 200
Хворий 23-х років надійшов до лікарні із черепно-мозковою травмою у важкому стані. Дихання характеризується судомним тривалим вдихом, який переривається коротким видихом. Для якого типу дихання це характерно? A 23-year-old patient was admitted to the hospital with a brain injury in a serious condition. Breathing is characterized by a convulsive long inhalation, which is interrupted by a short exhalation. For what type of breathing is this typical?

Куссмауля Kussmaul

Апнейстичне Apneistic

Гаспінг-дихання Gasping Breath

Чейн-Стокса Chain-Stokes

Біота Biota

39 / 200
Внаслідок дії на організм електричного струму міської електромережі впродовж 0,1 сек у напрямку 'права рука-голова'у постраждалого спостерігалась зупинка дихання. Вкажіть найбільш імовірний механізм цього ускладнення: As a result of the effect on the body of the electric current of the city power grid for 0.1 seconds in the direction of the 'right hand-head' of the victim, respiratory arrest was observed. Specify the most likely mechanism of this complication :

Параліч центрів вдиху Paralysis of inhalation centers

Тотальний параліч дихального центру Total paralysis of the respiratory center

Емоційний стрес Emotional stress

Рефлекторна зупинка дихання (больовий шок) Reflex respiratory arrest (pain shock)

Параліч дихальних м’язів Paralysis of respiratory muscles

40 / 200
В експерименті досліджували поріг сили подразника клітин різних тканин. Де він виявився найменшим? In the experiment, the threshold of the stimulus force of the cells of various tissues was studied. Where was it the smallest?

Мотонейрони спинного мозку Motor neurons of the spinal cord

Міоцити скелетного м’яза Skeletal muscle myocytes

Міоцити гладенького м’яза Smooth muscle myocytes

Залозисті клітини Glandular cells

Типові кардіоміоцити Typical cardiomyocytes

41 / 200
На ізольованому серці вивчалась швидкість проведення збудження у різних його ділянках. Де була виявлена найменша швидкість? On an isolated heart, the speed of conduction of excitation in different parts of it was studied. Where was the lowest speed found?

Міокард передсердь Atrial myocardium

Волокна Пуркін’є Purkinje fibers

Міокард шлуночків Myocardium of ventricles

Атріовентрикулярний вузол Atrioventricular node

Пучок Гіса His Bundle

42 / 200
Під час операції в печінці хворого виявлені дрібні міхурці малих розмірів з незначною кількістю рідини, які щільно прилягають один до одного. Який гельмінтоз виявився у хворого? During the operation in the patient's liver, small vesicles of small sizes with a small amount of liquid were found, which tightly adhere to each other. What kind of helminthiasis was found in the patient?

Опісторхоз Opistorchosis

Дікроцеліоз Dicroceliosis

Фасціольоз Fasciolosis

Клонорхоз Clonorchosis

Альвеококоз Alveococcosis

43 / 200
У хворого 70-ти років атеросклероз ускладнився тромбозом судин нижніх кінцівок, виникла гангрена пальців лівої стопи. Початок тромбоутворення, найбільш імовірно, пов’язаний з: In a 70-year-old patient, atherosclerosis was complicated by thrombosis of the vessels of the lower extremities, gangrene of the fingers of the left foot occurred. The onset of thrombus formation is most likely associated with:

Перетворенням фібриногену в фібрин Conversion of fibrinogen into fibrin

Зниженням синтезу гепарину Reduction of heparin synthesis

Перетворенням протромбіну в тром-бін By converting prothrombin into thrombin

Адгезією тромбоцитів By platelet adhesion

Активацією протромбінази Prothrombinase activation

44 / 200
Жінку госпіталізовано в клініку з симптомами гострого живота. При обстеженні виникла підозра на позаматкову вагітність. Яке з анатомічних утворень таза необхідно пропунктувати для підтвердження діагнозу? A woman was admitted to the clinic with symptoms of an acute abdomen. An ectopic pregnancy was suspected during the examination. Which of the pelvic anatomical formations must be identified to confirm the diagnosis?

Fossa ischiorectalis Fossa ischiorectalis

Excavatio vesicouterina Excavatio vesicouterina

Excavatio rectouterina Excavatio rectouterina

Excavatio rectovesicalis Excavatio rectovesicalis

Processus vaginalis peritonei Processus vaginalis peritonei

45 / 200
Дитина 10-ти років страждає на стафілококовий дерматит. Лікування бензилпеніциліном не дало результатів. Призначення комбінованого препарату пеніциліну з клавулановою кислотою дало швидке одужання. Яка причина позитивної дії цього препарату? A 10-year-old child suffers from staphylococcal dermatitis. Treatment with benzylpenicillin did not give results. The appointment of a combined penicillin with clavulanic acid resulted in a quick recovery. What is the reason for the positive effect of this drug ?

Гальмування аденозиндезамінази Inhibition of adenosine deaminase

Інактивація бета-лактамази Beta-lactamase inactivation

Блокада транслокази Translocase blockade

Гальмування транспептидази Inhibition of transpeptidase

Активація фосфодіестерази Activation of phosphodiesterase

46 / 200
У крові хворого виявлено підвищення активності ЛДГ 4,5, АлАТ, карбамоїлорнітинтрансферази. В якому органі можна передбачити розвиток патологічного процесу? An increase in the activity of LDH 4.5, ALT, carbamoylornithine transferase was detected in the patient's blood. In which organ can the development of the pathological process be predicted?

Сполучна тканина Connective tissue

Печінка (можливий гепатит) Liver (possible hepatitis)

Серцевий м’яз (можливий інфаркт міокарда) Myocardial muscle (possible myocardial infarction)

Скелетні м’язи Skeletal muscles

Нирки Kidneys

47 / 200
Апендикс, надісланий до патоморфологічного відділення після апендектомії, потовщений і збільшений у розмірах, серозна оболонка тьмяна, судини повнокровні, з просвіту відростка на розрізі виділяється рідина жовто-зеленого кольору. При якій формі апендициту виникають такі зміни? The appendix, sent to the pathology department after appendectomy, is thickened and enlarged, the serous membrane is dull, the vessels are full of blood, a yellow-green liquid is released from the lumen of the appendix on the section What form of appendicitis causes such changes?

Поверхневий катаральний Superficial catarrh

Гангренозний Gangrenous

Простий катаральний Simple catarrhal

Флегмонозний Phlegmonous

Апостематозний Apostematous

48 / 200
Внаслідок травми у чоловіка 40-ка років зруйновані задні корінці спинного мозку. Які розлади будуть спостерігатися в зоні іннервації цих корінців? As a result of an injury, a 40-year-old man has destroyed the posterior roots of the spinal cord. What disorders will be observed in the innervation zone of these roots?

Порушення функції гладеньких м’язів Smooth muscle dysfunction

Втрата температурної та вібраційної чутливості Loss of temperature and vibration sensitivity

Порушення функції посмугованих скелетних м’язів Striated skeletal muscle dysfunction

Втрата всіх видів чутливості Loss of all types of sensitivity

Втрата больової чутливості Loss of pain sensitivity

49 / 200
До клініки госпіталізована пацієнтка із скаргами на кровохаркання, пітливість. Рентгенологічно виявлено вогнище туберкульозу у верхній частці правої легені. Показана операція. Яку кількість сегментів можна видалити в складі верхньої частки правої легені? A female patient was admitted to the clinic with complaints of hemoptysis and sweating. X-ray revealed a tuberculosis focus in the upper lobe of the right lung. Surgery is indicated. How many segments can be removed in the upper lobe right lung?

5 5

3 3

1 1

2 2

4 4

50 / 200
У товстій кишці під час колоноскопії виявлено дефект слизової оболонки діаметром 3,5 см з нерівним горбистим дном, нерівними краями, що підвищуються над дном на 1,7 см; межа цього підвищення нечітка. Тканина дна і країв дефекту щільна, білувата, шари стінки кишки у цій ділянці не розрізняються. Встановіть макроскопічну форму пухлини: During colonoscopy, a mucosal defect with a diameter of 3.5 cm with an uneven bumpy bottom, uneven edges rising above the bottom by 1.7 cm was detected in the colon; the border of this increase is unclear. The tissue of the bottom and edges of the defect is dense, whitish, the layers of the intestinal wall in this area are not distinguishable. Establish the macroscopic form of the tumor:

Вузол Node

Інфільтрат Infiltrate

Інфільтративно-виразкова форма Infiltrative-ulcerative form

Кіста Cyst

Виразка Ulcer

51 / 200
У препараті в одній з судин мікроциркуляторного русла середня оболонка утворена 1-2 шарами гладеньких міоцитів, які розташовані поодинці і мають спіралеподібний напрямок. Зовнішня оболонка представлена тонким шаром пухкої волокнистої сполучної тканини. Вкажіть вид судини: In the preparation, in one of the vessels of the microcirculatory bed, the middle membrane is formed by 1-2 layers of smooth myocytes, which are located alone and have a spiral direction. The outer membrane is represented by a thin layer of loose fibrous connective tissue. Specify the type of vessel:

Посткапіляр Postcapillary

Капіляр Capillary

Артеріола Arteriole

Венула Venula

Артеріоловенулярний анастомоз Arteriovenular anastomosis

52 / 200
У хворої людини посилений рух води з кровоносних капілярів до тканин, що викликало їх позаклітинний набряк (збільшені розміри м’яких тканин кінцівок, печінки тощо). Зменшення якого параметру гомеостазу є найбільш імовірною причиною розвитку набряку? The sick person has an increased movement of water from the blood capillaries to the tissues, which caused their extracellular swelling (increased sizes of the soft tissues of the limbs, liver, etc.). Reduction of which parameter homeostasis is the most probable reason for the development of edema?

рН крові blood pH

Гематокрит Hematocrit

Онкотичний тиск плазми крові Oncotic pressure of blood plasma

Осмотичний тиск плазми крові Osmotic pressure of blood plasma

В’язкість крові Blood viscosity

53 / 200
У постраждалого в дорожній пригоді лікар виявив перелом лівої ключиці та порушення кровообігу в кінцівці (немає пульсації променевої артерії). Стиснення якої судини є причиною порушення кровообігу в кінцівці? The doctor discovered a fracture of the left clavicle and impaired blood circulation in the limb (there is no pulsation of the radial artery). Compression of which vessel is the cause of impaired blood circulation in the limb?

Пахвова вена Axillary vein

Підключична артерія Subclavian artery

Пахвова артерія Axillary artery

Підключична вена Subclavian vein

Хребтова артерія Vertebral artery

54 / 200
Хворого доставлено у медичний заклад в коматозному стані. Зі слів супроводжуючих вдалося з’ясувати, що він знепритомнів під час тренування на завершальному етапі марафонської дистанції. Який вид коми найімовірніше можна запідозрити у даного пацієнта? The patient was brought to the medical facility in a comatose state. According to the attendants, it was found that he fainted during training at the final stage of the marathon distance. What type of coma is most likely can this patient be suspected?

Гіперглікемічна Hyperglycemic

Гіпотиреоїдна Hypothyroid

Ацидотична Acidotic

Гіпоглікемічна Hypoglycemic

Печінкова Hepatic

55 / 200
У хворого коса пахвинна грижа. Яке анатомічне утворення стало слабким місцем передньої черевної стінки? The patient has an oblique inguinal hernia. What anatomical formation became the weak point of the anterior abdominal wall?

Стегнова ямка Femoral fossa

Латеральна пахвинна ямка Lateral inguinal fossa

Надміхурова ямка Scrotal fossa

Медіальна пахвинна ямка Medial inguinal fossa

Пахвинний трикутник Inguinal triangle

56 / 200
У хворих на тиреотоксикоз спостерігаються гіпертермія, булімія, зменшення маси тіла, що пов’язане з порушенням: In patients with thyrotoxicosis, hyperthermia, bulimia, and a decrease in body weight are observed, which is associated with the disorder:

Синтезу жирів Synthesis of fats

Циклу лимонної кислоти Cycle of citric acid

Розпаду АТФ ATP decay

в-окиснення жирних кислот b-oxidation of fatty acids

Спряження окислення і фосфорилю-вання Coupling of oxidation and phosphorylation

57 / 200
Людині внутрішньовенно ввели 0,5 л ізотонічного розчину лікарської речовини. Які з рецепторів насамперед прореагують на зміни водно-сольового балансу організму? A person intravenously injected 0.5 l of an isotonic solution of a medicinal substance. Which of the receptors will first of all react to changes in the body's water-salt balance?

Натрієві рецептори гіпоталамуса Sodium receptors of the hypothalamus

Волюморецептори порожнистих вен і передсердь Volume receptors of the vena cava and atria

Барорецептори дуги аорти Baroreceptors of the aortic arch

Осморецептори печінки Osmoreceptors of the liver

Осморецептори гіпоталамусу Osmoreceptors of the hypothalamus

58 / 200
Пацієнту 33 роки. Хворіє 10 років. Періодично звертається до лікаря зі скаргами на гострий біль у животі, судоми, порушення зору. У його родичів спостерігаються подібні симптоми. Сеча червоного кольору. Госпіталізований з діагнозом - гостра переміжна порфирія. Причиною захворювання може бути порушення біосинтезу такої речовини: The patient is 33 years old. He has been sick for 10 years. He periodically goes to the doctor with complaints of acute abdominal pain, convulsions, visual disturbances. Similar symptoms are observed in his relatives. Urine red in color. Hospitalized with a diagnosis of acute intermittent porphyria. The cause of the disease may be a violation of the biosynthesis of the following substance:

Жовчні кислоти Bile acids

Простагландини Prostaglandins

Колаген Collagen

Гем Ham

Інсулін Insulin

59 / 200
У хворої 45-ти років при електрокардіографічному обстеженні виявлено такі зміни: інтервал P — Q подовжений, при цьому випадає кожен другий або третій комплекс QRST. Яке саме порушення провідності серця спостерігається? In a 45-year-old patient, electrocardiographic examination revealed the following changes: the P — Q interval is prolonged, while every second or third QRST complex is lost. What exactly is the conduction disorder the heart is observed?

Атріовентрикулярна блокада ІІІ ступеня III degree atrioventricular block

Синоаурікулярна блокада Sino-auricular blockade

Атріовентрикулярна блокада повна Atrioventricular blockade is complete

Атріовентрикулярна блокада І ступеня Atrioventricular blockade of the 1st degree

Внутрішлуночкова блокада Intraventricular block

60 / 200
У вагітної жінки визначили групову належність крові. Реакцію аглютинації еритроцитів викликали стандартні сироватки I, III груп, та не викликала -сироватка II групи. Якою є група досліджуваної крові за системою АВ0? The blood group of a pregnant woman was determined. The erythrocyte agglutination reaction was caused by standard sera of groups I and III, but not -serum of group II. What is the group of the studied blood according to system AB0?

B(III), α B(III), α

A(II), β A(II), β

АВ (IV) AB (IV)

O(I), α, β O(I), α, β

- -

61 / 200
При розтині тіла померлого чоловіка 65-ти років, який страждав на захворювання легень, патологічний процес переважно був локалізований у бронхах, де при гістологічному дослідженні були чітко видні залози, хрящові острівці та багаторядний циліндричний миготливий епітелій. В яких бронхах відбулися зміни? At the autopsy of the body of a deceased 65-year-old man who suffered from lung disease, the pathological process was mainly localized in the bronchi, where glands were clearly visible during histological examination, cartilage islands and multi-rowed cylindrical ciliated epithelium. In which bronchi have changes occurred?

Малі бронхи Small bronchi

Головні бронхи Main bronchi

Термінальні бронхіоли Terminal bronchioles

Середні бронхи Medium bronchi

Великі бронхи Large bronchi

62 / 200
У юнака 18-ти років діагностована м’язова дистрофія. Підвищення в сироватці крові вмісту якої речовини найбільш імовірне при цій патології? An 18-year-old man is diagnosed with muscular dystrophy. What substance is most likely to increase in the blood serum with this pathology?

Креатин Creatine

Лактат Lactate

Міозин Myosin

Міоглобін Myoglobin

Аланін Alanine

63 / 200
У хворого в крові збільшена концентрація пірувату. Значна його кількість екскретується з сечею. Дефіцит якого вітаміну має місце у хворого? The patient has an increased concentration of pyruvate in the blood. A significant amount of it is excreted in the urine. What vitamin deficiency does the patient have?

E E

B6 B6

B2 B2

B1 B1

B3 B3

64 / 200
У 70-ті роки вчені встановили, що причиною важкої жовтяниці новонароджених є порушення зв’язування білірубіну в гепатоцитах. Яка речовина використовується для утворення кон’югату? In the 1970s, scientists established that the cause of severe jaundice in newborns is a violation of bilirubin binding in hepatocytes. What substance is used to form the conjugate?

Сірчана кислота Sulphuric acid

Сечова кислота Uric acid

Молочна кислота Lactic acid

Глюкуронова кислота Glucuronic acid

Піровиноградна кислота Pyrutaric acid

65 / 200
Хворому, що страждає на хронічний гепатит, для оцінки знешкоджуючої функції печінки було проведене навантаження бензоатом натрію. За виділенням якої речовини з сечею судять про знешкоджуючу функцію печінки? A patient suffering from chronic hepatitis was subjected to a load of sodium benzoate to assess the detoxifying function of the liver. Which substance is excreted in the urine to judge the detoxifying function of the liver?

Фенілоцтова кислота Phenylacetic acid

Валеріанова кислота Valeric acid

Щавелєва кислота Oxalic acid

Лимонна кислота Citric acid

Гіпурова кислота Hippuric acid

66 / 200
У хворого, який проходить курс лікувального голодування, нормальний рівень глюкози у крові підтримується головним чином за рахунок глюконеогенезу. З якої амінокислоти при цьому у печінці людини найбільш активно синтезується глюкоза? In a patient who undergoes a course of therapeutic fasting, the normal level of glucose in the blood is maintained mainly due to gluconeogenesis. From which amino acid is the most actively synthesized glucose in the human liver ?

Глутамінова кислота Glutamic acid

Лейцин Leucine

Аланін Alanine

Лізин Lysine

Валін Valin

67 / 200
В сечі новонародженого визначається цитрулін та високий рівень аміаку. Вкажіть, утворення якої речовини, наймовірніше, порушене у цього малюка: Citrulline and a high level of ammonia are detected in the urine of a newborn. Indicate the formation of which substance is most likely disturbed in this baby:

Аміак Ammonia

Сечовина Urea

Креатин Creatine

Сечова кислота Uric acid

Креатинін Creatinine

68 / 200
У здорової дорослої людини проводять зондування порожнин серця і великих судин. Де знаходиться зонд, якщо впродовж серцевого циклу зареєстровані зміни тиску від 0 до 120 мм рт.ст.? In a healthy adult, sounding of the heart cavity and large vessels is performed. Where is the probe located if pressure changes from 0 to 120 mm Hg are registered during the cardiac cycle?

Лівий шлуночок Left Ventricle

Правий шлуночок Right ventricle

Аорта Aorta

Передсердя Atrial

Легенева артерія Pulmonary artery

69 / 200
При дослідженні гнійних виділень з шийки матки бактеріоскопічно виявлено присутність грамнегативних бобоподібних диплококів, які знаходилися як в середині, так і поза лейкоцитами. Назвіть збудника гнійного запалення шийки матки: When examining purulent secretions from the cervix, bacterioscopically, the presence of gram-negative bean-like diplococci was found, which were both inside and outside leukocytes. Name the causative agent of purulent inflammation of the cervix:

Haemophilus vaginalis Haemophilus vaginalis

Neisseria gonorroeae Neisseria gonorroeae

Calymmatobacterium granulomatis Calymmatobacterium granulomatis

Trichomonas vaginalis Trichomonas vaginalis

Chlamidia trachomatis Chlamydia trachomatis

70 / 200
Дитина 1-го року відстає в розумовому розвитку від своїх однолітків. Ранком відзначаються блювання, судоми, непритомність. У крові - гіпоглікемія натще. З дефектом якого ферменту це пов’язане? A 1st-year-old child lags behind his peers in mental development. Vomiting, convulsions, fainting occur in the morning. Fasting hypoglycemia in the blood. Which enzyme defect is this associated with 'Yazane?

Аргіназа Arginase

Лактаза Lactase

Фосфорилаза Phosphorylase

Глікогенсинтетаза Glycogen synthetase

Сахараза Sucrase

71 / 200
Внаслідок черепно-мозкової травми у хворого розвинулись наступні симптоми: інтенційний тремор, дисметрія, адіадохокінез, дизартрія. Яка структура головного мозку ушкоджена? As a result of a brain injury, the patient developed the following symptoms: intention tremor, dysmetria, adiadochokinesis, dysarthria. What brain structure is damaged?

Чорна речовина Black Matter

Мозочок Cerebellum

Рухова кора Motor cortex

Бліда куля Pale Orb

Стріатум Striatum

72 / 200
У гістологічному препараті визначаються рецепторна зона сенсоепітеліального органа чуттів. Клітини даної зони лежать на базальній мембрані і включають наступні види: зовнішні та внутрішні сенсорні, зовнішні та внутрішні фалангові, стовбурові, зовнішні межові і зовнішні підтримуючі. Вкажіть, якому органу чуттів належить зазначена рецепторна зона: In the histological preparation, the receptor zone of the sensorepithelial sense organ is determined. The cells of this zone lie on the basement membrane and include the following types: external and internal sensory, external and internal phalangeal, trunk , external boundary and external supporting. Indicate to which sense organ the indicated receptor zone belongs:

Смаку Taste

Нюху Sniff

Зору Sight

Рівноваги Balances

Слуху Hearing

73 / 200
Хвора скаржиться на набряк ніг, посиніння шкіри, невеличкі виразки збоку латерального виростку. При обстеженні відмічено: припухлість, збільшення розмірів вен, утворення вузлів. З боку якої вени відмічається патологія? The patient complains of swelling of the legs, blue skin, small ulcers on the side of the lateral condyle. During the examination, swelling, an increase in the size of the veins, the formation of nodes are noted. On which side of the vein is the pathology?

V femoralis V femoralis

V saphena parva V saphena parva

V saphena magna V saphena magna

Vprofunda femoris Vprofunda femoris

V iliaca externa V iliaca externa

74 / 200
Хворому встановлений діагноз ураження голівки стегна ішемічного походження. Яка артерія ушкоджена? The patient has been diagnosed with a lesion of the femoral head of ischemic origin. Which artery is damaged?

Arteria illiaca externa Arteria illiaca externa

Arteria femoralis Arteria femoralis

Arteria umbilicalis Arteria umbilicalis

Ramus acetabularum A.obturatoriae Ramus acetabularum A. obturatoriae

Arteria profunda femoris Arteria profunda femoris

75 / 200
Чоловік 44-х років з інфарктом міокарда, помер від лівошлуночкової недостатності. На аутопсії: набряк легень, дрібнокраплинні крововиливи у серозних та слизових оболонках. Мікроскопічно: дистрофічні та некробіотичні зміни епітелію проксимальних канальців нирок, у печінці - центролобулярні крововиливи та осередки некрозу. Який з видів порушення кровообігу найбільш імовірний? A 44-year-old man with a myocardial infarction died of left ventricular failure. At autopsy: pulmonary edema, small droplet hemorrhages in the serous and mucous membranes. Microscopically: dystrophic and necrobiotic changes in the epithelium of the proximal tubules of the kidneys, in the liver - centrilobular hemorrhages and foci of necrosis. Which of the types of circulatory disorders is the most likely?

Хронічне недокрів’я Chronic anemia

Хронічне загальне венозне повнокров’я Chronic general venous congestion

Гостре недокрів’я Acute anemia

Артеріальна гіперемія Arterial hyperemia

Гостре загальне венозне повнокров’я Acute general venous congestion

76 / 200
У хворого гнійне запалення клиноподібної пазухи. В яку частину носової порожнини витікає гній? The patient has purulent inflammation of the sphenoid sinus. In which part of the nasal cavity does pus flow?

Meatus nasi inferior Meatus nasi inferior

Meatus nasi communis Meatus nasi communis

Meatus nasi superior Meatus nasi superior

Meatus nasi medius Meatus nasi medius

- -

77 / 200
Для підвищення результатів спортсмену рекомендували застосовувати препарат, який містить у собі карнітин. Який процес в найбільшому ступені активується карнітином? To improve results, the athlete was recommended to use a drug that contains carnitine. What process is activated to the greatest degree by carnitine?

Тканинне дихання Tissue respiration

Синтез стероїдних гормонів Synthesis of steroid hormones

Синтез ліпідів Synthesis of lipids

Синтез кетонових тіл Synthesis of ketone bodies

Транспорт жирних кислот до мітохондрій Transport of fatty acids to mitochondria

78 / 200
На ЕКГ пацієнта мають місце такі зміни: зубець P - нормальний, інтервал P — Q - вкорочений, шлуночковий комплекс QRST - розширений, зубець R -двогорбий або двофазний. Яка із форм аритмії має місце у даного пацієнта? The patient's ECG shows the following changes: the P wave is normal, the P — Q interval is shortened, the ventricular QRST complex is widened, the R wave is two-humped or biphasic. Which form of arrhythmia does this patient have?

Атріовентрикулярна блокада Atrioventricular block

Синдром Фредеріка (тріпотіння передсердь) Frederick syndrome (atrial flutter)

Синдром WPW (Вольфа-Паркінсона-Уайта) WPW (Wolf-Parkinson-White) syndrome

Миготлива аритмія Atrial fibrillation

Миготіння шлуночків Ventricular fibrillation

79 / 200
У хворого з жовтяницею встановлено: підвищення у плазмі крові вмісту загального білірубіну за рахунок непрямого (вільного), в калі та сечі - високий вміст стеркобіліну, рівень прямого (зв’язаного) білірубіну в плазмі крові в межах норми. Про який вид жовтяниці можна думати? In a patient with jaundice, it was established: an increase in the content of total bilirubin in the blood plasma due to indirect (free), in feces and urine - a high content of stercobilin, the level of direct (with bilirubin in the blood plasma is within the normal range. What type of jaundice can you think about?

Жовтяниця немовлят Infant Jaundice

Хвороба Жильбера Gilbert's disease

Гемолітична Hemolytic

Паренхіматозна (печінкова) Parenchymatous (hepatic)

Механічна Mechanical

80 / 200
Хвора 36-ти років страждає на колагеноз. Збільшення вмісту якого метаболіту найбільш імовірно буде встановлено у сечі? A 36-year-old patient suffers from collagenosis. An increase in the content of which metabolite is most likely to be detected in the urine?

Креатинін Creatinine

Уробіліноген Urobilinogen

Оксипролін Oxyproline

Індикан Indican

Сечовина Urea

81 / 200
У хворого на гіпертонічну хворобу виявлено високий рівень реніну в крові. Якому з гіпотензивних засобів слід віддати перевагу в цьому випадку? A patient with hypertension has a high level of renin in the blood. Which hypotensive agent should be preferred in this case?

Лізиноприл Lisinopril

Празозин Prazozin

Ніфедипін Nifedipine

Дихлотіазид Dichlorothiazide

Анаприлін Anaprilin

82 / 200
Причиною захворювання на пелагру може бути переважне харчування кукурудзою і зниження у раціоні продуктів тваринного походження. Відсутність у раціоні якої амінокислоти призводить до даної патології? The cause of pellagra disease can be a predominant diet of corn and a decrease in the diet of animal products. The lack of which amino acid in the diet leads to this pathology?

Триптофан Tryptophan

Гістидин Histidine

Ізолейцин Isoleucine

Фенілаланін Phenylalanine

Метіонін Methionine

83 / 200
У людини збільшена вентиляція легень внаслідок фізичного навантаження. Який з наведених показників зовнішнього дихання у неї значно більший, ніж у стані спокою? A person has increased ventilation of the lungs as a result of physical exertion. Which of the following indicators of external breathing is significantly greater than at rest?

Резервний об’єм видиху Reserve expiratory volume

Резервний об’єм вдиху Inspiratory reserve volume

Життєва ємність легень Vital lung capacity

Загальна ємність легень Total lung capacity

Дихальний об’єм Respiratory volume

84 / 200
Людина зробила максимально глибокий видих. Як називається об’єм повітря, що знаходиться в її легенях після цього? A person exhaled as deeply as possible. What is the volume of air in his lungs after that called?

Альвеолярний об’єм Alveolar volume

Залишковий об’єм Remaining Volume

Резервний об’єм видиху Reserve expiratory volume

Ємність вдиху Inhalation capacity

Функціональна залишкова ємність легень Functional residual lung capacity

85 / 200
Внаслідок активації іонних каналів зовнішньої мембрани збудливої клітини значно збільшився її потенціал спокою. Які канали були активовані? As a result of the activation of the ion channels of the outer membrane of the excitable cell, its resting potential increased significantly. Which channels were activated?

Швидкі кальцієві Fast calcium

Натрієві та кальцієві Sodium and calcium

Натрієві Sodium

Повільні кальцієві Slow calcium

Калієві Potassium

86 / 200
Робітник тваринницької ферми гостро захворів і при наростаючих явищах інтоксикації помер. При розтині тіла встановлено: селезінка збільшена, в’яла, на розрізі темновишневого кольору, зішкріб пульпи рясний. М’які мозкові оболонки на склепінні та основі мозку набряклі, просякнуті кров’ю, мають темно-червоний колір ('шапочка кардинала'). Мікроскопічно: серозно-геморагічне запалення оболонок і тканин головного мозку з руйнуванням стінок дрібних судин. Який найбільш імовірний діагноз? A worker on a livestock farm became acutely ill and died due to increasing symptoms of intoxication. An autopsy revealed that the spleen was enlarged, flaccid, dark cherry-colored on cross-section, pulp scraping was abundant. The soft meninges on the vault and base of the brain are swollen, soaked with blood, have a dark red color ('cardinal's cap'). Microscopically: serous-hemorrhagic inflammation of the meninges and tissues of the brain with destruction of the walls of small vessels. What is the most likely diagnosis ?

Чума Plague

Холера Cholera

Сибірка Anthrax

Бруцельоз Brucellosis

Туляремія Tularemia

87 / 200
При медичному обстеженні у військкоматі у юнака 18-ти років виявлено опускання головки надп’ясткової кістки, що призвело до плоскостопості. Зі слабкістю якої зв’язки стопи це пов’язано? During a medical examination at the Military Commissariat, an 18-year-old young man was found to have dropped the head of the metacarpal bone, which led to flat feet. The weakness of which ligaments of the foot caused this 'linked?

Підошвова п’ятково-човноподібна Sole heel-boat-shaped

Роздвоєна Forked

Надп’ятково-човноподібна Superheel-boat-shaped

Міжкісткова клино-кубоподібна Interosseous cuneiform

Підошвова клино-кубоподібна Sole wedge-cuboid

88 / 200
Після видалення у пацієнта 2/3 шлунка у крові зменшився вміст гемоглобіну, кількість еритроцитів, збільшилися розміри цих клітин крові. Дефіцит якого вітаміну призводить до таких змін у крові? After removing 2/3 of the patient's stomach, the hemoglobin content in the blood decreased, the number of erythrocytes increased, and the size of these blood cells increased. Which vitamin deficiency leads to such changes in the blood?

В12 B12

В6 B6

P P

PP PP

C C

89 / 200
У хворого 40-ка років при прогресуванні стафілококового гнійного періодонтиту виникло гнійне запалення кістково-мозкових просторів альвеолярного відростка, а потім тіла нижньої щелепи. Мікроскопічно кісткові балки витончені, вогнища некрозу, кісткові секвестри, оточені сполучнотканинною капсулою. Який найбільш імовірний діагноз? A 40-year-old patient developed purulent inflammation of the bone-marrow spaces of the alveolar process and then the body of the lower jaw during the progression of staphylococcal purulent periodontitis. Microscopically, the bone beams are thin, foci necrosis, bone sequestrations surrounded by a connective tissue capsule. What is the most likely diagnosis?

Гнійний періостит Suppurative periostitis

Гострий остеомієліт Acute osteomyelitis

Хронічний остеомієліт Chronic osteomyelitis

Хронічний фіброзний періостит Chronic fibrous periostitis

Пародонтома Periodontoma

90 / 200
При розтині тіла дитини, померлої від серцевої недостатності, виявлено: розширення порожнин шлуночків серця. Мікроскопічно в стромі міокарда повнокров’я, набряк, розповсюджені інфільтрати з гістіоцитів, лімфоцитів, нейтрофілів, еозинофілів. Який найбільш імовірний діагноз? At the autopsy of the body of a child who died of heart failure, it was found: expansion of the cavities of the ventricles of the heart. Microscopically, in the stroma of the myocardium, full blood, edema, widespread infiltrates of histiocytes, lymphocytes , neutrophils, eosinophils. What is the most likely diagnosis?

Вузликовий продуктивний міокардит Nodular productive myocarditis

Альтернативний мюкардит Alternative mucarditis

Проміжний продуктивний мюкардит Intermediate productive mycarditis

Дифузнопроміжний ексудативний міокардит Diffuse interstitial exudative myocarditis

Осередковий проміжний ексудативний міокардит Focus interstitial exudative myocarditis

91 / 200
Хвора 75-ти років доставлена до офтальмологічного відділення лікарні зі скаргами на погіршення зору. При об’єктивному дослідженні встановлена наявність пухлини мозку, що розташована в ділянці лівого зорового тракту. При цьому у хворої спостерігається випадіння поля зору в: A 75-year-old patient was brought to the ophthalmology department of the hospital with complaints of impaired vision. An objective examination revealed the presence of a brain tumor located in the area of the left optic tract At the same time, the patient has a loss of field of vision in:

Правих половинах сітківки обох очей Right halves of the retina of both eyes

Правих і лівих половинах сітківки правого ока Right and left halves of the retina of the right eye

Лівих половинах сітківки обох очей Left halves of the retina of both eyes

Правих і лівих половинах сітківок обох очей Right and left halves of the retinas of both eyes

Правих і лівих половинах сітківки лівого ока Right and left halves of the retina of the left eye

92 / 200
У хворого на ішемічну хворобу серця відзначається гіпертрофія міокарда, тахікардія, зниження ХОК. Який з механізмів є провідним в ушкодженні кардіоміоцитів у даному випадку? A patient with coronary heart disease has myocardial hypertrophy, tachycardia, and decreased IOC. Which of the mechanisms is the leading cause of cardiomyocyte damage in this case?

Дегідратація кардіоміоцитів Dehydration of cardiomyocytes

Пошкодження специфічних мембранних насосів Damage to specific membrane pumps

Збільшення числа а та β-адренорецепторів Втрата Mg2+ C. кардіоміоцитами Втрата Ca2+ D. кардіоміоцитами Increase in the number of α and β-adrenoceptors Loss of Mg2+ by C. cardiomyocytes Loss of Ca2+ by D. cardiomyocytes

93 / 200
Тварині внутрішньовенно ввели концентрований розчин хлориду натрію, що зумовило зниження реабсорбції іонів натрію у канальцях нирок. Внаслідок яких змін секреції гормонів це відбувається? The animals were intravenously injected with a concentrated solution of sodium chloride, which led to a decrease in the reabsorption of sodium ions in the tubules of the kidneys. What changes in the secretion of hormones does this occur?

Збільшення альдостерону Aldosterone Increase

Зменшення альдостерону Aldosterone reduction

Зменшення вазопресину Vasopressin reduction

Збільшення вазопресину Vasopressin Increase

Зменшення натрійуретичного фактора Reduction of natriuretic factor

94 / 200
У хворої 49-ти років відзначається обмеження довільних рухів у лівих кінцівках. Тонус м’язів у лівих руці та нозі підвищений за спастичним типом, посилені місцеві сухожилкові рефлекси, виявляються патологічні рефлекси. Який найбільш імовірний механізм призвів до розвитку м’язової гіпертонії та гіперрефлексії? A 49-year-old patient has limitation of voluntary movements in the left limbs. The muscle tone in the left arm and leg is increased according to the spastic type, local tendon reflexes are increased, pathological reflexes are detected. What is the most likely mechanism that led to the development of muscle hypertension and hyperreflexia?

Активація збуджуючих впливів з вогнища інсульту Activation of excitatory influences from the stroke focus

Зниження гальмівних низхідних впливів Decreasing inhibitory downstream effects

Гальмування мотонейронів кори головного мозку Inhibition of motoneurons of the cerebral cortex

Активація синаптичної передачі імпульсів Activation of synaptic impulse transmission

Активація мотонейронів внаслідок інсульту Motor neuron activation due to stroke

95 / 200
При виконуванні вправ на колоді гімнастка втратила рівновагу і впала. Із збудження, перш за все, яких рецепторів розпочнуться рефлекси, що забезпечать відновлення порушеної пози? While performing exercises on the floor, the gymnast lost her balance and fell. From excitement, first of all, which receptors will trigger the reflexes that will ensure the restoration of the broken posture?

Вестибулорецептори Vestibuloreceptors

Отолітові вестибулорецептори Otolith vestibuloreceptors

Пропріорецептори Proprioreceptors

Рецептори завитки Curl receptors

Ампулярні вестибулорецептори Ampullary vestibuloreceptors

96 / 200
В анотації до препарату вказано, що він містить антигени збудника черевного тифу, адсорбовані на стабілізованих еритроцитах барана. З якою метою використовують цей препарат? In the annotation to the drug, it is indicated that it contains antigens of the causative agent of typhoid, adsorbed on stabilized erythrocytes of a ram. For what purpose is this drug used?

Для виявлення антитіл в реакції зв’язування комплементу To detect antibodies in the complement binding reaction

Для серологічної ідентифікації збудника черевного тифу For serological identification of the causative agent of typhoid

Для виявлення антитіл в реакції непрямої гемаглютинації To detect antibodies in the indirect hemagglutination reaction

Для виявлення антитіл в реакції затримки гемаглютинації To detect antibodies in the hemagglutination delay reaction

Для виявлення антитіл в реакції Відаля To detect antibodies in the Vidal reaction

97 / 200
На гістологічному препараті видно позазародковий орган, який являє собою міхурець, зв’язаний з кишковою трубкою. Стінка його зсередини вистелена епітелієм, зовні утворена сполучною тканиною. На ранніх етапах ембріогенезу він виконує функцію кровотворного органа. Назвіть цей орган: On the histological preparation, an extra-embryonic organ is visible, which is a bladder connected to the intestinal tube. Its wall is lined from the inside with epithelium, and the outside is formed by connective tissue. In the early stages during embryogenesis, it performs the function of a hematopoietic organ. Name this organ:

Пуповина Umbilical Cord

Амніон Amnion

Плацента Placenta

Алантоїс Allantois

Жовтковий мішок Yolk Sac

98 / 200
Хворий був доставлений до лікарні в коматозному стані. В анамнезі - цукровий діабет. Об’єктивно: дихання Кусмауля, зниження артеріального тиску, у видихуваному повітрі запах ацетону. Після проведеної невідкладної терапії стан покращився. Який препарат було введено хворому? The patient was brought to the hospital in a comatose state. He has a history of diabetes. Objectively: Kussmaul's breathing, decreased blood pressure, the smell of acetone in the exhaled air. After the condition improved after emergency treatment. What drug was administered to the patient?

Ізадрин Izadrin

Інсулін Insulin

Адреналін Adrenaline

Глібенкламід Glibenclamide

Букаркам Bukarkam

99 / 200
У приймально-дiагностичне відділення доставили жінку 38-ми років з шлунковою кровотечею. Які зміни найбільш імовірні з боку крові через добу? A 38-year-old woman was brought to the reception and diagnostic department with gastric bleeding. What changes are most likely to occur in the blood after a day?

Еритроцитоз Erythrocytosis

Збільшення гематокритного числа Hematocrit increase

Лейкоцитоз Leukocytosis

Лейкопенія Leukopenia

Зменшення гематокритного числа Decreasing hematocrit number

100 / 200
При розтині трупа чоловіка 50-ти років виявлено наступні зміни: права легеня у всіх відділах помірно щільна, на розрізі тканина безповітряна, дрібнозерниста, сухувата. Вісцеральна плевра з нашаруванням фібрину сіро-коричневого кольору. Який найбільш імовірний діагноз? The autopsy of a 50-year-old man revealed the following changes: the right lung is moderately dense in all sections, the tissue is airless, fine-grained, dry on cross-section. Visceral pleura with layering gray-brown fibrin. What is the most likely diagnosis?

Крупозна пневмонія Croup pneumonia

Туберкульоз Tuberculosis

Бронхопневмонія Bronchopneumonia

Інтерстиціальна пневмонія Interstitial pneumonia

Пневмофіброз Pneumofibrosis

101 / 200
У людини звужені зіниці. Чим це зумовлено? A person has narrowed pupils. What causes this?

Зростання тонусу парасимпатичних центрів Increasing tone of parasympathetic centers

Зростання тонусу симпатичних центрів Increasing tone of sympathetic centers

Дія адреналіну Effect of adrenaline

Збільшення активності симпато-адреналової системи Increased activity of the sympatho-adrenal system

Дія норадреналіну Effect of norepinephrine

102 / 200
При розтині тіла жінки 50-ти років, померлої від серцевої недостатності, знайдено дифузне збільшення щитоподібної залози. Мікроскопічно: перетворення призматичного епітелію фолікулів у циліндричний, проліферація епітелію з утворенням сосочків, розрідження колоїду та лімфоплазмоцитарна інфільтрація строми; в серці - гіпертрофія лівого шлуночка, серозний набряк та лімфоїдна інфільтрація строми, в печінці - серозний набряк. Який з перелічених діагнозів найбільш імовірний? At the autopsy of a 50-year-old woman who died of heart failure, a diffuse enlargement of the thyroid gland was found. Microscopically: the transformation of the prismatic epithelium of the follicles into a cylindrical one, the proliferation of the epithelium with the formation papillae, thinning of the colloid and lymphoplasmacytic infiltration of the stroma; in the heart - hypertrophy of the left ventricle, serous edema and lymphoid infiltration of the stroma, in the liver - serous edema. Which of the listed diagnoses is the most probable?

Зоб Риделя Riedel's goiter

Ендемічний зоб Endemic goiter

Дифузний токсичний зоб Diffuse toxic goiter

Гострий тиреоїдит Acute thyroiditis

Зоб Хашімото Hashimoto's goiter

103 / 200
Пацієнт через 15 діб після повернення з багатомісячного плавання в районах Середземномор’я та Західної Африки відчув слабкість, головний біль, періодичні підвищення температури. Лікар запідозрив у хворого малярію. Який із перерахованих методів є найбільш адекватним в діагностиці даного захворювання? 15 days after returning from months of sailing in the Mediterranean and West Africa, the patient experienced weakness, headache, and periodic fever. The doctor suspected the patient of malaria. Which of the listed methods is the most adequate in the diagnosis of this disease?

Алергічний Allergic

Мікробіологічний Microbiological

Мікроскопічний Microscopic

Серологічний Serological

Біологічний Biological

104 / 200
У вагітної жінки взяли кров для підтвердження клінічного діагнозу 'токсоплазмоз'. Яка із перерахованих серологічних реакцій має діагностичне значення? Blood was taken from a pregnant woman to confirm the clinical diagnosis of 'toxoplasmosis'. Which of the listed serological reactions has diagnostic significance?

Реакція нейтралізації Neutralization reaction

Реакція гемадсорбції Hemadsorption reaction

Реакція гальмування гемаглютинації Hemagglutination inhibition reaction

Реакція аглютинації Agglutination reaction

Реакція зв’язування комплементу Complement binding reaction

105 / 200
У здорових батьків, спадковість яких не обтяжена, народилась дитина з чисельними вадами розвитку. Цитогенетичний аналіз виявив в соматичних клітинах дитини трисомію за 13-ю хромосомою (синдром Патау). З яким явищем пов’язане народження такої дитини? A child with multiple developmental disabilities was born to healthy parents whose heredity is not burdened. Cytogenetic analysis revealed trisomy on the 13th chromosome (Patau syndrome) in the child's somatic cells What phenomenon is associated with the birth of such a child?

Рецесивна мутація Recessive mutation

Хромосомна мутація Chromosome mutation

Соматична мутація Somatic mutation

Порушення гаметогенезу Disruption of gametogenesis

Домінантна мутація Dominant mutation

106 / 200
У хворого, який скаржиться на поліурію і полідипсію, знайдено цукор в сечі. Вміст цукру в плазмі крові у нормі. З чим пов’язаний механізм глюкозурії у хворого? Sugar was found in the urine of a patient who complains of polyuria and polydipsia. The content of sugar in the blood plasma is normal. What is the mechanism of glucosuria in the patient?

Iнсулiнорезистентнiсть рецепторів клітин Insulin resistance of cell receptors

Порушення реабсорбції глюкози в канальцях нефрону Disturbance of glucose reabsorption in nephron tubules

Гіперпродукція глюкокортикоїдів наднирниками Hyperproduction of glucocorticoids by the adrenal glands

Недостатня продукція інсуліну підшлунковою залозою Insufficient production of insulin by the pancreas

Порушення фільтрації глюкози в клубочковому відділі нефрону Disturbance of glucose filtration in the glomerular part of the nephron

107 / 200
Під час об’єктивного обстеження хворого з діагнозом: атеросклеротичний міокардіосклероз, лікар встановив феномен дефіциту пульсу. При якій формі порушення серцевого ритму спостерігається такий феномен? During an objective examination of a patient with a diagnosis of atherosclerotic myocardiosclerosis, the doctor established the phenomenon of a pulse deficiency. In what form of heart rhythm disturbance is this phenomenon observed?

Передсердно-шлуночковий ритм Atrial-ventricular rhythm

Синусова екстрасистолія Sinus extrasystole

Миготлива аритмія Atrial fibrillation

Ідіовентрикулярний ритм Idioventricular rhythm

Брадикардія Bradycardia

108 / 200
На препараті представлено орган, покритий сполучнотканинною капсулою, від якої відходять трабекули. В органі можна розрізнити кіркову речовину, де містяться лімфатичні вузлики та мозкову речовину, представлену тяжами лімфоїдних клітин. Який орган представлений на препараті? The preparation shows an organ covered with a connective tissue capsule from which trabeculae depart. In the organ, you can distinguish the cortical substance, which contains lymph nodes, and the medullary substance, represented by strands of lymphoid cells What organ is represented on the drug?

Тимус Thymus

Мигдалики Tonsils

Червоний кістковий мозок Red bone marrow

Лімфатичний вузол Lymph node

Селезінка Spleen

109 / 200
До лікаря звернулася жінка 25-ти років зі скаргами на дисменорею та безпліддя. При обстеженні виявлено: зріст жінки 145 см, недорозвинені вторинні статеві ознаки, на шиї крилоподібні складки. При цитологічному дослідженні в соматичних клітинах не виявлено тілець Барра. Який діагноз встановив лікар? A 25-year-old woman came to the doctor with complaints of dysmenorrhea and infertility. The examination revealed: the woman's height is 145 cm, underdeveloped secondary sexual characteristics, wing-like folds on the neck Barr corpuscles were not detected in somatic cells during cytological examination. What diagnosis did the doctor make?

Синдром Шерешевського-Тернера Shereshevsky-Turner syndrome

Синдром трисомії Х Trisomy X syndrome

Синдром Морріса Morris Syndrome

- -

Синдром Клайнфельтера Klinefelter syndrome

110 / 200
У хворої внаслідок запалення порушена ендокринна функція фолікулярних клітин фолікулів яєчника. Синтез яких гормонів буде пригнічений? The endocrine function of the follicular cells of the ovarian follicles is disturbed in the patient as a result of inflammation. The synthesis of which hormones will be inhibited?

Фолікулостимулюючий гормон Follicle-stimulating hormone

Прогестерон Progesterone

Лютропін Lutropin

Фолістатин Follistatin

Естрогени Estrogens

111 / 200
Хворому на гострий інфаркт міокарда у комплексній терапії було призначено гепарин. Через деякий час після введення даного препарату з’явилась гематурія. Який антагоніст гепарину необхідно ввести хворому для усунення даного ускладнення? A patient with an acute myocardial infarction was prescribed heparin in complex therapy. Some time after the administration of this drug, hematuria appeared. What heparin antagonist should be administered to the patient to eliminate this complication?

Вікасол Vikasol

Неодикумарин Neodicoumarin

Амінокапронова кислота Aminocaproic acid

Фібриноген Fibrinogen

Протаміну сульфат Protamine sulfate

112 / 200
У хворого на хронічну серцеву недостатність, незважаючи на терапію кардіотонічними засобами і тіазидовим діуретиком, зберігаються набряки і виникла загроза асциту. Який препарат слід призначити для підсилення діуретичного ефекту застосованих ліків? In a patient with chronic heart failure, despite therapy with cardiotonic agents and a thiazide diuretic, edema persists and there is a threat of ascites. What drug should be prescribed to enhance the diuretic effect of the drugs used ?

Клопамід Clopamide

Манітол Mannitol

Амілорид Amiloride

Спіронолактон Spironolactone

Фуросемід Furosemide

113 / 200
Пацієнта турбують поліурія (7 л на добу) і полідипсія. При обстеженні не виявлено ніяких розладів вуглеводного обміну. Дисфункція якої ендокринної залози може бути причиною даних порушень? The patient is concerned about polyuria (7 liters per day) and polydipsia. The examination did not reveal any disorders of carbohydrate metabolism. Dysfunction of which endocrine gland could be the cause of these disorders?

Нейрогіпофіз Neurohypophysis

Острівці підшлункової залози Pancreatic islets

Аденогіпофіз Adenohypophysis

Мозкова речовина наднирників Adrenal brain substance

Кора наднирників Adrenal cortex

114 / 200
У жінки 20-ти років під час медичного огляду при пальпації в молочній залозі виявлено щільний інкапсульований вузол діаметром 1,0 см. Результат післяопераційного біопсійного дослідження: розростання сполучної тканини навколо протоків молочної залози та залозисті елементи різного діаметру, що не утворюють часточок, без ознак клітинного атипізму. Яка відповідь патологоанатома? During a medical examination of a 20-year-old woman, palpation revealed a dense encapsulated nodule with a diameter of 1.0 cm in the mammary gland. Postoperative biopsy result: growth of connective tissue around the ducts of the mammary gland and glandular elements of various diameters that do not form lobules, without signs of cellular atypism. What is the pathologist's answer?

Фіброзний рак Fibrous cancer

Метастаз раку Cancer metastasis

Фіброма Fibroma

Аденома Adenoma

Фіброаденома Fibroadenoma

115 / 200
У хворого із швидко наростаючою внутрішньомозковою гіпертензією діагностована пухлина мозку. Під час операції видалена пухлина тім’яно-скроневої частки, м’якої консистенції, на розрізі строкатого вигляду. Гістологічно пухлина побудована з поліморфних гіперхромних клітин з утворенням псевдорозеток та великої кількості судин, ділянками некрозів і крововиливами. Який найбільш імовірний діагноз? A brain tumor was diagnosed in a patient with rapidly increasing intracerebral hypertension. During the operation, a tumor of the parietal-temporal lobe, soft consistency, with a variegated appearance was removed. Histologically, the tumor is composed of polymorphic hyperchromic cells with the formation of pseudorosettes and a large number of vessels, areas of necrosis and hemorrhages. What is the most likely diagnosis?

Гліобластома Glioblastoma

Менінгіома Meningioma

Астроцитома Astrocytoma

Олігодендрогліома Oligodendroglioma

Арахноїдендотеліома Arachnoidendothelioma

116 / 200
У хворого з кровотечею розвинулась гостра ниркова недостатність, що спричинила смерть. На аутопсії макроскопічно: нирки збільшені з широким блідо-рожевим кірковим шаром, різко відмежованим від темно-червоних пірамід. Мікроскопічно: відсутність ядер епітелію звивистих канальців, тубулорексіс, венозний застій, ядра клітин судинних клубочків та прямих канальців збережені. Яка патологія нирок розвинулась у хворого? A patient with bleeding developed acute renal failure, which caused death. At autopsy macroscopically: the kidneys are enlarged with a wide pale pink cortical layer sharply demarcated from dark red pyramid. Microscopically: the absence of convoluted tubule epithelial nuclei, tubulorhexis, venous stasis, the nuclei of vascular glomeruli cells and straight tubules are preserved. What kidney pathology has developed in the patient?

Нефроз Nephrosis

Пієлонефрит Pyelonephritis

Гломерулонефрит Glomerulonephritis

Інфаркт Heart attack

Некронефроз Necronephrosis

117 / 200
З метою масового обстеження студентів на носійство S.aureus перед виробничою практикою у дитячому відділенні клінічної лікарні було використано елективне середовище з метою отримання чистої культури цього збудника. Яке з перерахованих середовищ було використано? For the purpose of mass screening of students for S.aureus carriers before production practice in the children's department of a clinical hospital, an elective medium was used to obtain a pure culture of this pathogen. Which of the following environment was used?

Кров’яний телуритовий агар Blood tellurite agar

Середовище Вільсона-Блера Wilson-Blair environment

М’ясо-пептонний агар Meat peptone agar

Середовище Ендо Environment Endo

Жовтково-сольовий агар Yolk-salt agar

118 / 200
В крові хворого виявлено високий вміст галактози, концентрація глюкози знижена. Відмічена розумова відсталість, помутніння кришталика. Яке захворювання має місце? In the patient's blood, a high content of galactose was detected, the concentration of glucose was reduced. Mental retardation, opacification of the lens was noted. What disease is there?

Галактоземія Galactosemia

Фруктоземія Fructosemia

Лактоземія Lactosemia

Стероїдний діабет Steroid diabetes

Цукровий діабет Diabetes

119 / 200
Під час бійки у чоловіка виникла зупинка серця внаслідок сильного удару у верхню ділянку передньої черевної стінки. Який із зазначених механізмів спричинив зупинку серця? During the fight, the man suffered a cardiac arrest as a result of a strong blow to the upper part of the front abdominal wall. Which of the following mechanisms caused the cardiac arrest?

Симпатичні безумовні рефлекси Sympathetic unconditioned reflexes

Симпатичні умовні рефлекси Sympathetic conditioned reflexes

Парасимпатичні умовні рефлекси Parasympathetic conditioned reflexes

Парасимпатичні безумовні рефлекси Parasympathetic unconditioned reflexes

Периферичні рефлекси Peripheral reflexes

120 / 200
У хворого, що надійшов до хірургічного відділення з ознаками гострого апендициту, виявлені наступні зміни білої крові: загальна кількість лейкоцитів - 16 • 109 /л. Лейкоцитарна формула: б.- 0, е.- 2%, ю.- 2%, п.- 8%, с.- 59%, л.-25%, м.- 4%. Як класифікуються зазначені зміни? The following changes in white blood were detected in a patient who was admitted to the surgical department with signs of acute appendicitis: the total number of leukocytes is 16 • 109 /l. Leukocyte formula: b .- 0, e.- 2%, yu.- 2%, n.- 8%, s.- 59%, l.-25%, m.- 4%. How are the specified changes classified?

Лейкемоїдна реакція за нейтрофільним типом Leukemoid reaction by neutrophilic type

Нейтрофілія з гіперрегенеративним зсувом вліво Neuprophilia with hyperregenerative shift to the left

Нейтрофілія з зсувом вправо Neutrophilia with shift to the right

Нейтрофілія з дегенеративним зсувом вліво Neutrophilia with a degenerative shift to the left

Нейтрофілія з регенеративним зсувом вліво Neutrophilia with a regenerative shift to the left

121 / 200
У людини визначили частоту серцевих скорочень за пульсом. Вона дорівнює 120/хв. Якою при цьому є тривалість серцевого циклу? A person's heart rate was determined by pulse. It is equal to 120/min. What is the duration of the heart cycle?

0,5 с 0.5 s

0,7 с 0.7 s

1,0 с 1.0 s

0,9 с 0.9 s

0,8 с 0.8 s

122 / 200
У хворого із вираженим пневмосклерозом після перенесеного інфільтративного туберкульозу легень розвинулась дихальна недостатність. До якого патогенетичного типу вона відноситься? A patient with pronounced pneumosclerosis developed respiratory failure after a history of infiltrative pulmonary tuberculosis. What pathogenetic type does it belong to?

Дисрегуляційний Disregulatory

Апнеїстичний Apneic

Рефлекторний Reflex

Рестриктивний Restrictive

Обструктивний Obstructive

123 / 200
Після перенесеної стрептококової інфекції у чоловіка діагностовано гострий гломерулонефрит. Найбільш імовірно, що ураження базальної мембрани ниркових тілець виникає внаслідок алергічної реакції такого типу: After a streptococcal infection, a man was diagnosed with acute glomerulonephritis. It is most likely that the damage to the basal membrane of the renal corpuscles is caused by an allergic reaction of the following type:

Анафілактична Anaphylactic

Цитотоксична Cytotoxic

Сповільнена Delayed

Імунокомплексна Immunocomplex

Стимулююча Stimulating

124 / 200
До лікаря звернулась мати з приводу поганого самопочуття дитини - відсутність апетиту, поганий сон, дратівливість. При біохімічному дослідженні в крові виявлено відсутність ферменту глюкоцереброзидази. Для якої патології це характерно? The mother consulted a doctor about the child's poor health - lack of appetite, poor sleep, irritability. A biochemical examination revealed the absence of the glucocerebrosidase enzyme in the blood. What pathology is this characteristic of ?

Хвороба Гірке Bitter disease

Хвороба Німана-Піка Niemann-Pick disease

Хвороба Гоше Gauche disease

Хвороба Тея-Сакса Tay-Sachs disease

Хвороба Помпе Pompe's disease

125 / 200
Дитина 9-ми місяців харчується штучними сумішами, які не збалансовані за вмістом вітаміну B6. У дитини спостерігається пелагроподібний дерматит, судоми, анемія. Розвиток судом може бути пов’язаний з порушенням утворення: A 9-month-old child is fed with artificial mixtures that are not balanced in terms of vitamin B6 content. The child has pellagra-like dermatitis, convulsions, anemia. The development of convulsions may be due to associated with a violation of formation:

ГАМК GABA

Дофаміну Dopamine

Серотоніну Serotonin

ДОФА DOFA

Гістаміну Histamine

126 / 200
Внаслідок отруєння неякісною їжею хворому було промито шлунок розчином калію перманганату. Який механізм дії цього засобу? Due to poor-quality food poisoning, the patient's stomach was washed with potassium permanganate solution. What is the mechanism of action of this remedy?

Вивільнення йоду Iodine release

Порушення синтезу ферментів дихального ланцюга Disruption of synthesis of respiratory chain enzymes

Вивільнення хлору Chlorine release

Вивільнення атомарного кисню Release of atomic oxygen

Порушення мембран бактерій Violation of bacterial membranes

127 / 200
Чоловік протягом 3-х років працював в одній із африканських країн. Через місяць після переїзду до України звернувся до офтальмолога зі скаргами на біль в очах, набряки повік, сльозоточивість і тимчасове послаблення зору. Під кон’юнктивою ока були виявлені гельмінти розмірами 30-50 мм, які мали видовжене ниткоподібне тіло. Який найбільш імовірний діагноз? The man worked in one of the African countries for 3 years. A month after moving to Ukraine, he turned to an ophthalmologist with complaints of eye pain, swelling of the eyelids, lacrimation and temporary weakening of vision. Helminths measuring 30-50 mm were found under the conjunctiva of the eye, which had an elongated thread-like body. What is the most likely diagnosis?

Ентеробіоз Enterobiosis

Філяріоз Filariasis

Аскаридоз Ascariasis

Трихоцефальоз Trichocephalus

Дифілоботріоз Diphyllobotriosis

128 / 200
У пацієнта з бронхіальною астмою за допомогою шкірних алергічних проб встановлено сенсибілізацію алергеном тополиного пуху. Який фактор імунної системи відіграє вирішальну роль в розвитку цього імунопатологічного стану? A patient with bronchial asthma was diagnosed with poplar down allergen sensitization using skin allergy tests. What factor of the immune system plays a decisive role in the development of this immunopathological condition?

IgM IgM

IgG IgG

Т-лімфоцити T-lymphocytes

IgE IgE

IgD IgD

129 / 200
У пацієнта діагностовано компресійний перелом поперекового хребця. При цьому різко збільшилась кривизна лордозу поперекового відділу хребта. Ушкодженням якої зв’язки може супроводжуватись така зміна кривизни хребта? The patient was diagnosed with a compression fracture of the lumbar vertebra. At the same time, the curvature of the lordosis of the lumbar spine increased sharply. Damage to which ligament can be accompanied by such a change in the curvature of the spine?

Задня поздовжня Back Longitudinal

Передня поздовжня Front longitudinal

Міжостиста Interstellar

Клубово-поперекова Iliolumbar

Жовта Yellow

130 / 200
При дослідженні підозрілих м’ясних продуктів (сосиски), що мали характерних гнилісний запах, було виділено рухливі грамнегативні паличковидні мікроорганізми, що добре росли на МПА з ефектом 'роїння'. При посіві в конденсаційну воду мікроорганізми на поверхні середовища утворювали наліт димчасто-блакитного кольору. Який мікроорганізм міг спричинити гнилісний розпад даного продукту? During the study of suspicious meat products (sausages) that had a characteristic rotten smell, motile gram-negative rod-shaped microorganisms were isolated that grew well on MPA with a 'swarming' effect '. When inoculated into condensation water, microorganisms formed a smoky-blue coating on the surface of the medium. What microorganism could have caused the putrefaction of this product?

Протей Proteus

Кишкова паличка Escherichia coli

Сальмонели Salmonella

Шигели дизентерії Shigel dysentery

Холерний вібріон Vibrio cholerae

131 / 200
До лабораторії доставлено кров хворого з підозрою на черевний тиф для проведення серологічного дослідження. Реакцію Відаля поставив недостатньо кваліфікований лаборант, який обмежився використанням тільки Ота Н-діагностикумів із сальмонел тифу. Які ще діагностикуми слід було використати для правильної постановки реакції Відаля? Blood from a patient with suspected typhoid fever was delivered to the laboratory for serological testing. Vidal's reaction was given by an insufficiently qualified laboratory technician who limited himself to using only Ota H-diagnostics from salmonella typhus . What other diagnostics should have been used to correctly describe Vidal's reaction?

Холери та дизентерії Cholera and dysentery

Висипного та поворотного тифів Eruptive and relapsing typhus

Паратифів А та В Paratypes A and B

Еритроцитарні О- та Н-діагностикуми Erythrocyte O- and H-diagnostics

К та Vi діагностикуми сальмонел тифу K and Vi diagnostics of salmonella typhus

132 / 200
У дитини виявлена схильність до ожиріння, яка є результатом діатезу Назвіть вид діатезу, при якому найчастіше може розвинутись ожиріння: The child has a tendency to obesity, which is the result of a diathesis. Name the type of diathesis in which obesity can most often develop:

- -

Астенічний Asthenic

Ексудативно-катаральний Exudative-catarrhal

Лімфатико-гіпопластичний Lymphatic-hypoplastic

Нервово-артритичний Nervous-arthritic

133 / 200
Інозитолтрифосфати в тканинах організму утворюються в результаті гідролізу фосфатидилінозитолдифосфатів і відіграють роль вторинних посередників (месенджерів) в механізмі дії гормонів. Їхній вплив у клітині спрямований на: Inositol triphosphates in body tissues are formed as a result of hydrolysis of phosphatidylinositol diphosphates and play the role of secondary mediators (messengers) in the mechanism of hormone action. Their influence in the cell is aimed at:

Вивільнення іонів кальцію з клітинних депо Release of calcium ions from cellular depots

Гальмування протеїнкінази С Inhibition of protein kinase C

Активацію аденілатциклази Activation of adenylate cyclase

Активацію протеїнкінази А Activation of protein kinase A

Гальмування фосфодіестерази Inhibition of phosphodiesterase

134 / 200
У хворого спостерігаються геморагії, в крові знижена концентрація протромбіну. Недостатність якого вітаміну призвела до порушення синтезу цього фактору згортання крові? The patient has hemorrhages, the concentration of prothrombin in the blood is reduced. The lack of which vitamin led to a violation of the synthesis of this blood coagulation factor?

D D

C C

K K

A A

E E

135 / 200
При отруєнні невідомим препаратом у пацієнта спостерігались сухість слизової оболонки рота та розширення зіниць. З яким впливом пов’язана дія цього препарату? When poisoned by an unknown drug, the patient experienced dryness of the mucous membrane of the mouth and dilation of the pupils. What is the effect of this drug?

Стимуляція адренорецепторів Stimulation of adrenoceptors

Стимуляція H-холінорецепторів Stimulation of H-cholinergic receptors

Стимуляція M-холінорецепторів Stimulation of M-cholinergic receptors

Блокада M-холінорецепторів Blockade of M-cholinergic receptors

Блокада адренорецепторів Adrenoceptor blockade

136 / 200
У відпочиваючого в санаторії у результаті сонячного опіку на шкірі спини утворилися міхурці, заповнені світлою рідиною, оточені зоною гіперемії, болісні. Який з перерахованих механізмів лежить в основі формування ексудації у вогнищі запалення? In a sanatorium vacationer, as a result of a sunburn, blisters filled with a light liquid, surrounded by a zone of hyperemia, painful formed on the skin of the back. Which of the listed mechanisms underlies the formation of exudation in the focus of inflammation?

Зменшення виведення рідини з тканини Decreased removal of fluid from tissue

Збільшення колоїдно-осмотичного тиску в тканині Increase in colloid-osmotic pressure in the tissue

Збільшення кількості лізосомальних ферментів Increase in the number of lysosomal enzymes

Зменшення рівня кейлонів у тканині Decreasing keylon level in fabric

Еміграція лейкоцитів з судин Emigration of leukocytes from blood vessels

137 / 200
У лабораторії особливо небезпечних інфекцій проводиться мікроскопічне дослідження патологічного матеріалу від хворого з підозрою на чуму, забарвленого за Гінсом-Буррі. Яку властивість збудника дозволяє визначити даний метод? In the laboratory of especially dangerous infections, a microscopic study of pathological material from a patient suspected of plague, stained according to Ginsom-Burry, is carried out. What property of the causative agent can be determined by this method?

Кислотостійкість Acid resistance

Наявність зерен волютину Presence of volutin grains

Спороутворення Spore formation

Лугостійкість Alkali resistance

Капсулоутворення Encapsulation

138 / 200
Внаслідок вираженого зниження концентрації кальцію в плазмі крові у дитини 2-х років виникли тетанічні скорочення дихальних і глоткових м’язів. Зниження секреції якого гормону може бути причиною цього? As a result of a pronounced decrease in the concentration of calcium in the blood plasma, a 2-year-old child developed tetanic contractions of the respiratory and pharyngeal muscles. A decrease in the secretion of which hormone can be the reason for this?'

Паратгормон Parathyroid hormone

Соматотропін Somatotropin

Альдостерон Aldosterone

Кортизол Cortisol

Тиреокальцитонін Thyrocalcitonin

139 / 200
У хворого з лихоманкою та висипкою на шкірі після обстеження за допомогою серологічних реакцій поставлений діагноз фасціольоз. Було встановлено, що хворий заразився шляхом споживання сирої води з річки. Яка стадія життєвого циклу фасціоли інвазійна для людини? A patient with fever and a rash on the skin was diagnosed with fasciolosis after examination using serological reactions. It was established that the patient became infected by consuming raw water from the river. What stage the life cycle of fasciola is invasive for humans?

Адолескарій Adolescarius

Яйце Egg

Фіна Fina

Мірацидій Miracidium

Метацеркарій Metacercarium

140 / 200
До лікаря-педіатра звернулись батьки новонародженої дитини зі скаргами на виділення рідини (сечі) в ділянці пупка. Яка вроджена вада у дитини? Parents of a newborn child turned to a pediatrician with complaints about discharge of fluid (urine) in the navel area. What is the child's congenital defect?

Дивертикул Меккеля Meckel's diverticulum

Розщеплення сечівника Splitting of the ureter

Пупкова грижа Umbilical hernia

Пахвинна грижа Inguinal hernia

Незрощена сечова протока Unfused ureter

141 / 200
При харчовому отруєнні виділена культура анаеробних грампозитивних спороутворюючих паличок. До якого виду, найбільш імовірно, відноситься виділений збудник? In case of food poisoning, a culture of anaerobic gram-positive spore-forming bacilli was isolated. To what species is the isolated pathogen most likely?

Proteus vulgaris Proteus vulgaris

C. perfringens C. perfringens

P.mirabilis P.mirabilis

Esherichia coli Esherichia coli

Vibrio parahemolyticus Vibrio parahemolyticus

142 / 200
До бактеріологічної лабораторії доставлені блювотні маси хворого з підозрою на холеру. З матеріалу приготований препарат 'висяча крапля'. Який метод мікроскопії буде використаний для виявлення збудника по його рухливості? Vomiting masses of a patient suspected of cholera were delivered to the bacteriological laboratory. A 'hanging drop' preparation was prepared from the material. What microscopy method will be used to detect the causative agent by its mobility?

Електронна Electronic

Люмінесцентна Fluorescent

Фазово-контрастна Phase-contrast

Імунна електронна Immune Electronic

Шерстна Wooly

143 / 200
У результаті радіаційного випромінювання ушкоджені стовбурові гемопоетичні клітини. Утворення яких клітин сполучної тканини буде порушено? As a result of the radiation, stem hematopoietic cells are damaged. The formation of which connective tissue cells will be disturbed?

Макрофаги Macrophages

Фібробласти Fibroblasts

Адіпоцити Adipocytes

Періцити Pericytes

Меланоцити Melanocytes

144 / 200
У потерпілого з травмою голови у скроневій ділянці діагностовано епідуральну гематому. Яка з артерій найімовірніше пошкоджена? A victim with a head injury in the temporal area was diagnosed with an epidural hematoma. Which of the arteries is most likely damaged?

Поверхнева скронева Superficial temporal

Середня мозкова Average cerebral

Середня оболонкова Medium shell

Задня вушна Back auricle

Передня оболонкова Front Shell

145 / 200
У дитини спостерігається затримка фізичного та розумового розвитку, глибокі порушення з боку сполучної тканини внутрішніх органів, у сечі виявлено кератансульфати. Обмін яких речовин порушений? The child has a delay in physical and mental development, deep disorders of the connective tissue of internal organs, keratan sulfates were detected in the urine. What substances are the metabolism disturbed?

Гіалуронова кислота Hyaluronic acid

Еластин Elastin

Фібронектин Fibronectin

Колаген Collagen

Глікозаміноглікани Glycosaminoglycans

146 / 200
У хворої 45-ти років невроз, що проявляється дратівливістю, безсонням, немотивованою тривогою. Який лікарський засіб усуне всі симптоми? A 45-year-old patient has a neurosis manifested by irritability, insomnia, and unmotivated anxiety. What medicine will eliminate all symptoms?

Кофеїн-бензоат натрію Caffeine sodium benzoate

Пірацетам Piracetam

Леводопа Levodopa

Діазепам Diazepam

Екстракт валеріани Valerian extract

147 / 200
У хворого після вживання недоброякісної їжі розвинулася діарея. На наступний день у нього знизився артеріальний тиск, з’явились тахікардія, екстрасистолія. pH крові складає 7,18. Ці порушення є наслідком розвитку: The patient developed diarrhea after consuming poor-quality food. The next day, his blood pressure decreased, tachycardia, extrasystole appeared. Blood pH is 7.18. These violation is a consequence of development:

Негазового ацидозу Nongaseous acidosis

Газового алкалозу Gas alkalosis

Негазового алкалозу Nongaseous alkalosis

Газового ацидозу Gas acidosis

Метаболічного алкалозу Metabolic alkalosis

148 / 200
При ендоскопії шлунка виявлена атрофія слизової оболонки. Мікроскопічно в біоптаті виявлена пухлина, побудована з ланцюжків атипових епітеліальних клітин, розташованих серед тяжів грубоволокнистої сполучної тканини; строма значно переважає над паренхімою. Який найбільш імовірний діагноз? During endoscopy of the stomach, mucosal atrophy was detected. Microscopically, the biopsy revealed a tumor made of chains of atypical epithelial cells located among strands of coarse fibrous connective tissue; the stroma significantly prevails over the parenchyma . What is the most likely diagnosis?

Медулярний рак Medullary cancer

Солідний рак Solid cancer

Дрібноклітинний рак Small cell cancer

Аденокарцинома Adenocarcinoma

Скирозний рак Canker cancer

149 / 200
Слизова оболонка трубчастого органу вкрита перехідним епітелієм, утворює поздовжні складки. М’язова оболонка складається з двох шарів у верхній половині і трьох у нижній. Який це орган? The mucous membrane of a tubular organ is covered with transitional epithelium, forming longitudinal folds. The muscle membrane consists of two layers in the upper half and three in the lower half. What kind of organ is this?

Стравохід Esophagus

Сечовий міхур Bladder

Пряма кишка Rectum

Сечовід Ureter

Маткова труба Fallopian tube

150 / 200
До лікарні доставлений хворий з отруєнням інсектицидом - ротеноном. Яка ділянка мітохондріального ланцюга переносу електронів блокується цією речовиною? A patient with rotenone insecticide poisoning was brought to the hospital. Which part of the mitochondrial electron transport chain is blocked by this substance?

Цитохром С-оксидаза Cytochrome C oxidase

НАДН-коензим Q-редуктаза NADH coenzyme Q-reductase

Сукцинат-коензим Q-редуктаза Succinate coenzyme Q-reductase

АТФ -синтетаза ATP synthetase

Коензим Q-цитохром С-редуктаза Coenzyme Q-cytochrome C-reductase

151 / 200
У хворого, який довготривало приймав преднізолон, в результаті відміни препарату виникло загострення захворювання, зниження артеріального тиску, слабкість. З чим можна зв’язати ці прояви? A patient who has been taking prednisone for a long time, as a result of withdrawal of the drug, there was an exacerbation of the disease, a decrease in blood pressure, weakness. What can be associated with these manifestations?

Гіперпродукція АКТГ Hyperproduction of ACTH

Виникнення недостатності кори наднирників Emergence of adrenal cortex insufficiency

Кумуляція препарату Cumulation of drug

Звикання до препарату Addiction to the drug

Сенсибілізація до препарату Sensitization to the drug

152 / 200
У дитини 7-ми років, хворої на ангіну, був взятий матеріал (мазок з поверхні мигдаликів) і засіяний на кров’яний агар. Через добу виросли колонії стрептококів, навколо яких середовище стало прозорим. Наявність якого фактору патогенності у збудника виявило це дослідження? A 7-year-old child with tonsillitis had material taken (smear from the surface of the tonsils) and inoculated on blood agar. A day later, streptococci colonies grew , around which the environment became transparent. The presence of which factor of pathogenicity in the causative agent was detected by this study?

Ендотоксин Endotoxin

Гемолізин Hemolysin

Бета -лактамаза Beta-lactamase

Лейкоцидин Leukocidin

Нейрамінідаза Neuraminidase

153 / 200
У хворого на алкоголізм почався алкогольний психоз із вираженим психомоторним збудженням. Який препарат з групи нейролептиків слід призначити для швидкої допомоги? A patient with alcoholism has started an alcoholic psychosis with pronounced psychomotor agitation. Which drug from the group of neuroleptics should be prescribed for emergency care?

Натрію бромід Sodium bromide

Аміназин Aminazine

Резерпін Reserpin

Фторотан Fluorotan

Діазепам Diazepam

154 / 200
При огляді дитини 4-х років зі скаргами на загальну слабкість, біль у горлі та утруднене ковтання лікар запідозрив дифтерію та направив матеріал до бактеріологічної лабораторії. На яке диференціально-діагностичне поживне середовище слід засіяти матеріал для виділення збудника дифтерії? When examining a 4-year-old child with complaints of general weakness, sore throat, and difficulty swallowing, the doctor suspected diphtheria and sent the material to the bacteriological laboratory. On which differential should the diagnostic nutrient medium be sown with material for isolation of the causative agent of diphtheria?

Середовище Ендо Environment Endo

Кров’яно-телуритовий агар Blood Tellurite Agar

Середовище Сабуро Sabour Environment

Середовище Плоскірєва Environment of Ploskirev

Середовище Левенштейна-Йєнсена Lewenstein-Jensen Environment

155 / 200
Серед антиатеросклеротичних препаратів, що застосовуються з метою профілактики та лікування атеросклерозу, є левостатин. Він діє шляхом: Among the antiatherosclerotic drugs used for the prevention and treatment of atherosclerosis is levostatin. It works by:

Усіма наведеними шляхами All given paths

Стимулювання екскреції холестерину з організму Stimulation of cholesterol excretion from the body

Активації метаболізму холестерину Activations of cholesterol metabolism

Пригнічення всмоктування холестерину в кишечнику Inhibition of cholesterol absorption in the intestine

Гальмування біосинтезу холестерину Cholesterol biosynthesis inhibition

156 / 200
У хворого під час ендоскопічного обстеження на слизовій оболонці шлунка виявлено кілька ерозій. Який із зазначених лікарських засобів міг спричинити таке ускладнення? During the endoscopic examination of the patient, several erosions were found on the mucous membrane of the stomach. Which of the mentioned medicines could have caused such a complication?

Дигоксин Digoxin

Диклофенак-натрій Diclofenac sodium

Фамотидин Famotidine

Глауцин Glaucin

Діазепам Diazepam

157 / 200
При лабораторному дослідженні у хворого виявили стеаторею. Вкажіть фермент, недостатність дії якого призвела до виникнення цього симптому: During a laboratory examination, steatorrhea was detected in the patient. Specify the enzyme, the insufficient action of which led to the occurrence of this symptom:

Лактаза Lactase

Хімотрипсин Chymotrypsin

Ліпаза Lipase

Амілаза Amylase

Пепсин Pepsin

158 / 200
Людина впродовж тривалого часу вживала їжу, бідну на метіонін, внаслідок чого у неї спостерігалися розлади функції нервової та ендокринної систем. Це може бути наслідком порушення синтезу: A person for a long time consumed food poor in methionine, as a result of which he had disorders of the function of the nervous and endocrine systems. This may be the result of a violation of synthesis:

Жирних кислот Fatty acids

Тироніну Tyronin

Пірувату Pyruvate

Глюкагону Glucagon

Адреналіну Adrenaline

159 / 200
Жінка 45-ти років декілька років хворіє на системний червоний вовчак у легкій формі. При прогресуванні захворювання (з’явився міокардит) їй призначили преднізолон як імуносупресор. Через 2 місяці прийому у хворої виникла шлункова кровотеча. Яка найбільш імовірна її причина? A 45-year-old woman has been suffering from mild systemic lupus erythematosus for several years. When the disease progressed (myocarditis appeared), she was prescribed prednisone as an immunosuppressant. After 2 In the month of admission, the patient developed gastric bleeding. What is the most likely cause?

Зменшення згортання крові Decreased blood coagulation

Ульцерогенна дія Ulcerogenic action

Підвищення артеріального тиску Increased blood pressure

Подальше прогресування захворювання Further progression of the disease

Збудження ЦНС CNS excitation

160 / 200
У новонародженої дитини виявлено вроджені вади розвитку травної системи, що пов’язано з дією тератогенних факторів на початку вагітності. На якій з зародкових листків подіяв тератоген? Congenital malformations of the digestive system were detected in a newborn child, which is associated with the action of teratogenic factors at the beginning of pregnancy. On which of the germ layers did the teratogen act?

Ендодерма і мезодерма Endoderm and Mesoderm

Усі листки All leaves

Ендодерма Endoderm

Мезодерма Mesoderm

Ектодерма Ectoderm

161 / 200
Дитина під час гри порізала ногу осколком скла і була направлена у поліклініку для введення протиправцевої сироватки. З метою попередження розвитку анафілактичного шоку лікувальну сироватку вводили за Безредкою. Який механізм лежить в основі подібного способу гіпосенсибілізації організму? The child cut his leg with a piece of glass while playing and was sent to the polyclinic for administration of anti-tetanus serum. In order to prevent the development of anaphylactic shock, the therapeutic serum was administered according to Bezredka. What is the mechanism in the basis of this method of hyposensitization of the body?

Блокування синтезу медіаторів у тучних клітинах Blocking the synthesis of mediators in mast cells

Зв’язування фіксованих на тучних клітинах IgE Mat cell-fixed IgE binding

Стимуляція імунологічної толерантності до антигену Stimulation of immunological tolerance to antigen

Стимуляція синтезу антиген-специфічних IgG Stimulation of antigen-specific IgG synthesis

Зв’язування рецепторів до IgE на тучних клітинах IgE receptor binding on mast cells

162 / 200
До токсикологічного відділення надійшов хворий з симптомами гострого отруєння сполуками ртуті. Який препарат необхідно призначити у якості антидоту? A patient with symptoms of acute mercury poisoning came to the toxicological department. What drug should be prescribed as an antidote?

Баррол Barrol

Трифтазин Triftazine

Унітіол Unithiol

Нейромедин Neuromedin

Плантаглюцин Plantaglucin

163 / 200
Недостатність в організмі лінолевої та ліноленової кислот призводить до ушкоджень шкіри, випадіння волосся, сповільненого загоювання ран, тромбоцитопенії, зниження опірності до інфекційних захворювань. Порушення синтезу яких речовин найімовірніше зумовлює вказані симптоми? Insufficiency of linoleic and linolenic acids in the body leads to skin damage, hair loss, delayed wound healing, thrombocytopenia, reduced resistance to infectious diseases. Violation of the synthesis of which substances most likely causes indicated symptoms?

Інтерлейкіни Interleukins

Катехоламіни Catecholamines

Кортикостероїди Corticosteroids

Інтерферони Interferons

Ейкозаноїди Eicosanoids

164 / 200
У жінки, що тривало приймала антибіотики з приводу кишкової інфекції, розвинулось ускладнення з боку слизової порожнини рота у вигляді запального процесу і білого нальоту, у якому під час бактеріологічного дослідження були виявлені дріжджеподібні грибки Candida albicans. Який з перерахованих препаратів показаний для лікування цього ускладнення? A woman who had been taking antibiotics for an intestinal infection for a long time developed a complication from the mucous membrane of the mouth in the form of an inflammatory process and a white plaque, in which during a bacteriological examination yeast-like fungi Candida albicans were detected. Which of the listed drugs is indicated for the treatment of this complication?

Фуразолідон Furazolidone

Бісептол Biseptol

Поліміксин Polymixin

Флуконазол Fluconazole

Тетрациклін Tetracycline

165 / 200
У хворого в лейкограмі: лейкоцити - 14 • 109 /л; мієлобласти - 71%, промієлоцити, мієлоцити, метамієлоцити - 0%, паличкоядерні нейтрофіли - 6%, сегментоядерні -13%; лімфоцити - 7%, моноцити - 3%. Яка патологія у хворого? In the patient's leukogram: leukocytes - 14 • 109 /l; myeloblasts - 71%, promyelocytes, myelocytes, metamyelocytes - 0%, rod-shaped neutrophils - 6%, segmentonuclear - 13%, lymphocytes - 7%, monocytes - 3%. What is the patient's pathology?

Лімфобластний лейкоз Lymphoblastic leukemia

Нейтрофільний лейкоцитоз Neutrophil leukocytosis

Мієлобластний лейкоз Myeloblastic leukemia

Хронічний мієлолейкоз Chronic myelogenous leukemia

Хронічний лімфолейкоз Chronic lymphocytic leukemia

166 / 200
Під час розтину тіла померлої жінки 54-х років з діагнозом 'вузликовий періартеріїт', мікроскопічно були виявлені такі змін в артеріолах: мукоїдне і фібриноїдне набухання, фібриноїдний некроз стінок, дифузна лімфогістіоцитарна інфільтрація, зміни ендотеліоцитів. Назвіть вид запалення в артеріолах: During the autopsy of a dead woman aged 54 with a diagnosis of nodular periarteritis, the following changes in arterioles were microscopically detected: mucoid and fibrinoid swelling, fibrinoid wall necrosis , diffuse lymphohistiocytic infiltration, changes in endotheliocytes. Name the type of inflammation in arterioles:

Дифтеритичне запалення Diphtheritic inflammation

Фібринозне запалення Fibrinous inflammation

Дифузне ексудативне запалення Diffuse exudative inflammation

Гостре не імунне запалення Acute non-immune inflammation

Гостре імунне запалення Acute immune inflammation

167 / 200
При дослідженні гостроти слуху в коваля виявили втрату слуху на 50% у діапазоні низьких частот і майже нормальну гостроту слуху в діапазоні високих частот. Порушення яких структур слухової системи призвело до такого стану? When examining the hearing acuity of a blacksmith, they found a 50% hearing loss in the low frequency range and almost normal hearing acuity in the high frequency range. Violation of which structures of the auditory system led to such a state?

Середня частина кортієвого органу Middle part of the organ of Corti

Кортієв орган - ближче до овального віконця Organ of Corti - closer to the oval window

М’язи середнього вуха Middle ear muscles

Кортієв орган - ближче до гелікотреми Organ of Corti - closer to helicotrema

Барабанна перетинка Eardrum

168 / 200
При відборі для ревакцинації вакциною БЦЖ у школяра поставлено пробу Манту, яка виявилася негативною. Результат проби свідчить про такі особливості імунітету до туберкульозу: During selection for revaccination with the BCG vaccine, a Mantoux test was performed on a schoolboy, which turned out to be negative. The test result indicates the following features of immunity to tuberculosis:

Відсутність гуморального імунітету Lack of humoral immunity

Наявність гуморального імунітету Availability of humoral immunity

Відсутність клітинного імунітету Absence of cellular immunity

Відсутність антитоксичного імунітету Lack of antitoxic immunity

Наявність клітинного імунітету Presence of cellular immunity

169 / 200
Людина вийшла з кондиціонованого приміщення назовні, де немає вітру, температура повітря +38oC, вологість 64%. За рахунок якого механізму буде здійснюватися віддача тепла організмом за цих умов? A person left the air-conditioned room outside, where there is no wind, the air temperature is +38oC, the humidity is 64%. At the expense of what mechanism will the heat transfer by the body be carried out under these conditions?'

Теплопроведення Heat conduction

Кондукція Conduction

Теплорадіація Thermal radiation

Випаровування поту Evaporation of sweat

Конвекція Convection

170 / 200
Робітниця хімічного підприємства внаслідок порушення правил безпечної роботи зазнала токсичної дії азотистої кислоти та нітритів, які викликають дезамінування цитозину в молекулі ДНК. Який фермент ініціює ланцюг репараційних процесів? As a result of violating the rules of safe work, a worker at a chemical plant was exposed to the toxic effects of nitric acid and nitrites, which cause the deamination of cytosine in the DNA molecule. What enzyme initiates the chain of repair processes?

Оротидилмонофосфат-декарбоксилаза Orotidyl monophosphate decarboxylase

Цитидинтрифосфатсинтетаза Cytidine triphosphate synthetase

ДНК-залежна-РНК-полімераза DNA-dependent-RNA-polymerase

Уридин-ДНК-глікозидаза Uridine DNA glycosidase

Тимідилатсинтаза Thymidylate synthase

171 / 200
У пацієнта з передозуванням наркотичної речовини відсутня свідомість, гіпотермія, гіпотензія, стійкий міоз, порушення дихання по типу Чейн-Стокса. Який функціональний антагоніст необхідно призначити для забезпечення виживання пацієнта? A patient with a drug overdose has unconsciousness, hypothermia, hypotension, persistent miosis, Cheyne-Stokes respiratory failure. What functional antagonist should be prescribed to ensure the patient's survival ?

Омепразол Omeprazole

Мезатон Mesaton

Нітразепам Nitrazepam

Налоксон Naloxone

Етимізол Etimizole

172 / 200
Хворий надійшов до клініки зі скаргами на загальну слабкість, порушення сну. Шкіра має жовтий колір. У крові: збільшена кількість прямого білірубіну, жовчних кислот. Кал ахолічний. Для якого стану характерні ці зміни? The patient came to the clinic with complaints of general weakness, sleep disturbances. The skin is yellow. In the blood: an increased amount of direct bilirubin, bile acids. Acholic stool. For What condition are these changes characteristic of?

Гемолітична жовтяниця Hemolytic Jaundice

Синдром Жільбера Gilbert syndrome

Механічна жовтяниця Mechanical jaundice

Надпечінкова жовтяниця Suprahepatic jaundice

Хронічний холецистит Chronic cholecystitis

173 / 200
У жінки 22-х років через 5 годин після вживання морепродуктів на шкірі тулуба та дистальних відділів кінцівок з’явились маленькі сверблячі папули, які частиною зливаються між собою. Через добу висипка самовільно зникла. Назвіть механізм гіперчутливості, що полягає в основі даних змін: In a 22-year-old woman, 5 hours after eating seafood, small itchy papules appeared on the skin of the trunk and distal parts of the limbs, which partially merge with each other. day the rash spontaneously disappeared. Name the mechanism of hypersensitivity underlying these changes:

Імунокомплексна гіперчутливість Immune complex hypersensitivity

Антитілоопосередкований клітинний цитоліз Antibody-mediated cellular cytolysis

Клітинна цитотоксичність Cellular cytotoxicity

Атопія (місцева анафілаксія) Atopy (local anaphylaxis)

Системна анафілаксія Systemic anaphylaxis

174 / 200
Хворий на гіпертонічну хворобу разом з безсольовою дієтою та з антигіпертензивними засобами, довгий час приймав гідрохлортіазид, що зумовило порушення електролітного балансу. Яке порушення внутрішнього середовища виникло у хворого? A hypertensive patient, along with a salt-free diet and antihypertensive drugs, has been taking hydrochlorothiazide for a long time, which caused a violation of the electrolyte balance. What violation of the patient's internal environment occurred?

Гіпермагніємія Hypermagnesemia

Гіпохлоремічний алкалоз Hypochloremic alkalosis

Збільшення об’єму циркулюючої крові Increase in circulating blood volume

Метаболічний ацидоз Metabolic acidosis

Гіперкаліємія Hyperkalemia

175 / 200
При авторадіографічному дослідженні епітелію тонкої кишки було виявлено, що його повне оновлення відбувається протягом 3-х діб за рахунок проліферації малодиференційованих клітин. Вкажіть їх локалізацію: During the autoradiographic study of the epithelium of the small intestine, it was found that its complete renewal occurs within 3 days due to the proliferation of poorly differentiated cells. Specify their localization:

Основа ворсинок Base of villi

Верхівка ворсинок Top of villi

Бічна поверхня ворсинок Lateral surface of villi

Власна пластинка слизової оболонки Own plate of the mucous membrane

Дно крипт Bottom of Crypts

176 / 200
Пасажири автобуса у спекотну погоду попросили відкрити люки. Який шлях тепловіддачі при цьому зростає найбільше? Bus passengers in hot weather asked to open the hatches. Which path of heat transfer increases the most?

Випромінювання та теплопроведення Radiation and heat conduction

Випромінювання Radiation

Теплопроведення Heat conduction

Випаровування поту Evaporation of sweat

Конвекція Convection

177 / 200
У хворого через добу після апендектомії при аналiзi крові виявили нейтрофільний лейкоцитоз з регенеративним зсувом вліво. Який найбільш імовірний механізм розвитку абсолютного лейкоцитозу у периферичній крові хворого? One day after appendectomy, a patient's blood analysis revealed neutrophilic leukocytosis with a regenerative shift to the left. What is the most likely mechanism of the development of absolute leukocytosis in the patient's peripheral blood?

Зменшення руйнування лейкоцитів Decreased destruction of leukocytes

Активація імунітету Activation of immunity

Посилення лейкопоезу Increasing leukopoiesis

Перерозподіл лейкоцитів в організмі Redistribution of leukocytes in the body

Уповільнення еміграції лейкоцитів у тканині Slowing down of emigration of leukocytes in the tissue

178 / 200
При загостренні ревматоїдного артриту хворому, в анамнезі якого супутній хронічний гастрит, призначений целекоксиб. Чим обумовлено зменшення побічної дії препарату на травний тракт? For an exacerbation of rheumatoid arthritis, a patient with a history of accompanying chronic gastritis is prescribed celecoxib. What is the reason for the reduction of side effects of the drug on the digestive tract?

Переважаюче пригнічення циклооксигенази-1 Predominant inhibition of cyclooxygenase-1

Переважаюче пригнічення циклооксигенази-2 Predominant inhibition of cyclooxygenase-2

Пригнічення фосфоліпази А2 Inhibition of phospholipase A2

Пригнічення фосфодіестерази Inhibition of phosphodiesterase

Переважаюча стимуляція аденіла-тциклази Predominant stimulation of adenyl-tcyclase

179 / 200
В експерименті на кролі встановлено, що об’єм кисню, який споживається головним мозком за 1 хвилину, дорівнює об’єму CO2 , який виділяється клітинами мозку в кров. Це свідчить, що у клітинах головного мозку має місце: In an experiment on a rabbit, it was established that the volume of oxygen consumed by the brain in 1 minute is equal to the volume of CO2 released by brain cells into the blood. This shows that in the cells of the brain there is:

Окислення жирів Oxidation of fats

Окислення вуглеводів Oxidation of carbohydrates

Гіпоксія Hypoxia

Окислення білків Protein oxidation

Гіпокапнія Hypocapnia

180 / 200
Хворому поставлено діагноз газова гангрена. Після ідентифікації збудника досліджуваний матеріал необхідно знищити. Який метод слід використати? The patient was diagnosed with gas gangrene. After identification of the causative agent, the research material must be destroyed. What method should be used?

Стерилізація парою під тиском Steam sterilization under pressure

Пастеризація Pasteurization

Кип’ятіння Boiling

Стерилізація текучою парою Steam sterilization

Тиндалізація Tyndalization

181 / 200
Фенілкетонурія - це захворювання, яке зумовлено рецесивним геном, що локалізується в аутосомі. Батьки є гетерозиготами за цим геном. Вони вже мають двох хворих синів і одну здорову доньку. Яка імовірність, що четверта дитина, яку вони очікують, народиться теж хворою? Phenylketonuria is a disease caused by an autosomal recessive gene. The parents are heterozygous for this gene. They already have two sick sons and one healthy daughter. What is the probability that the fourth child they are expecting will also be born sick?

0% 0%

50% 50%

25% 25%

75% 75%

100% 100%

182 / 200
До косметолога звернулася пацієнтка зі скаргами на появу чорних цяток на обличчі. Після обстеження було встановлено, що поява цяток пов’язана з порушенням виділення секрету сальних залоз. Який тип секреції характерний для цих залоз? A patient turned to a cosmetologist with complaints about the appearance of black spots on her face. After the examination, it was established that the appearance of spots is associated with a violation of the secretion of the sebaceous glands. What type secretion characteristic of these glands?

Мікроапокриновий Microapocrine

Мерокриновий та мікроапокриновий Merocrine and microapocrine

Мерокриновий Merocrine

Макроапокриновий Macroapocrine

Голокриновий Holocrine

183 / 200
У хворого 40-ка років ознаки гірської хвороби: запаморочення, задишка, тахікардія, рН крові - 7,50, pCO2 -30 мм рт.ст., зсув буферних основ +4 ммоль/л. Яке порушення кислотноосновного стану має місце? A 40-year-old patient has signs of mountain sickness: dizziness, shortness of breath, tachycardia, blood pH - 7.50, pCO2 -30 mm Hg, shift buffer bases +4 mmol/l. What violation of the acid-base state is taking place?

Негазовий ацидоз Nongaseous acidosis

Видільний ацидоз Excretory acidosis

Негазовий алкалоз Nongaseous alkalosis

Газовий алкалоз Gas alkalosis

Газовий ацидоз Gas acidosis

184 / 200
Після ремонту автомобіля в закритому приміщенні при працюючому двигуні у чоловіка з’явилися задишка, запаморочення, акроціаноз, частота дихання 24-26/хв. Газовий склад крові: pO2 - 60 мм рт.ст., pCO2 - 30 мм рт.ст.; у крові наявний карбоксигемоглобін. Про який вид гіпоксії можна думати? After repairing the car in a closed room with the engine running, the man developed shortness of breath, dizziness, acrocyanosis, respiratory rate 24-26/min. Blood gas composition: pO2 - 60 mm Hg, pCO2 - 30 mm Hg; carboxyhemoglobin is present in the blood. What kind of hypoxia can you think about?

Респіраторна Respiratory

Циркуляторна Circulator

Гіпоксична Hypoxic

Гемічна Chemical

Тканинна Fabric

185 / 200
У клітині в гранулярній ЕПС відбувається етап трансляції, при якому спостерігається просування і-РНК щодо рибосоми. Амінокислоти з’єднуються пептидними зв’язками в певній послідовності - відбувається біосинтез поліпептиду. Послідовність амінокислот у поліпептиді буде відповідати послідовності: In a cell in the granular EPS, a stage of translation occurs, during which the advancement of i-RNA relative to the ribosome is observed. Amino acids are connected by peptide bonds in a certain sequence - biosynthesis occurs polypeptide. The sequence of amino acids in the polypeptide will correspond to the sequence:

Антикодонів т-РНК T-RNA anticodons

Нуклеотидів т-РНК T-RNA nucleotides

Нуклеотидів р-РНК R-RNA nucleotides

Кодонів і-РНК I-RNA codons

Антикодонів р-РНК Anticodons of p-RNA

186 / 200
При обстеженні у хворого виявлене порушення чутливості шкіри в ділянці передньої поверхні шиї. Який нерв уражений? During the examination, the patient was found to have a skin sensitivity disorder in the area of the front surface of the neck. Which nerve is affected?

Надключичні Supraclavicular

Малий потиличний Small occipital

Поперечний нерв шиї Transverse neck nerve

Шийна петля Neck loop

Великий вушний Big ear

187 / 200
Хворий на хронічну серцеву недостатність тривалий час приймав препарат з групи серцевих глікозидів. У нього з’явились нудота, слабкість, екстрасистолія. Яке явище обумовило розвиток цих симптомів? A patient with chronic heart failure took a drug from the group of cardiac glycosides for a long time. He developed nausea, weakness, extrasystole. What phenomenon caused the development of these symptoms?

Функціональна кумуляція Functional cumulation

Звикання Addiction

Лікарська залежність Drug addiction

Матеріальна кумуляція Material accumulation

Ідіосинкразія Idiosyncrasy

188 / 200
При ультразвуковому обстеженні дитини був виявлений дивертикул Меккеля. Аномалію розвитку якої кишки діагностовано? Meckel's diverticulum was detected during the ultrasound examination of the child. What intestinal abnormality was diagnosed?

Ободова Bezel

Порожня Empty

Сліпа Blind

Клубова Club

Сигмоподібна Sigma

189 / 200
До реанімаційного відділення в тяжкому стані, без свідомості надійшов пацієнт. Діагностовано передозування барбітуратів, які спричинили феномен тканинної гіпоксії. На якому рівні відбулося блокування електронного транспорту? A patient came to the intensive care unit in a serious condition, unconscious. An overdose of barbiturates was diagnosed, which caused the phenomenon of tissue hypoxia. At what level did the blocking of electronic transport occur?

Убіхінон Ubiquinone

АТФ -синтаза ATP synthase

Цитохромоксидаза Cytochrome oxidase

Цитохром b - цитохром c1 Cytochrome b - cytochrome c1

НАДН-коензимQ-редуктаза NADH-coenzyme Q-reductase

190 / 200
Встановлено, що в клітинах організмів відсутні мембранні органели та їх спадковий матеріал не має нуклеосомної організації. Що це за організми? It was established that the cells of organisms lack membrane organelles and their hereditary material does not have a nucleosome organization. What kind of organisms are these?

Еукаріоти Eukaryotes

Аскоміцети Ascomycetes

Прокаріоти Prokaryotes

Віруси Viruses

Найпростіші The easiest

191 / 200
У пацієнта діагностований первинний туберкульоз легень. Призначення якого протитуберкульозного засобу з групи антибіотиків, що порушують синтез РНК мікобактерій, є бажаним? The patient is diagnosed with primary pulmonary tuberculosis. Which antituberculosis drug from the group of antibiotics that disrupt the synthesis of RNA of mycobacteria is recommended?

Канаміцину сульфат Kanamycin sulfate

Етамбутол Etambutol

ПАСК PASK

Рифампіцин Rifampicin

Піразинамід Pyrazinamide

192 / 200
Хлопчик на другому році життя став часто хворіти на респіраторні захворювання, стоматити, гнійничкові ураження шкіри. Навіть невеликі пошкодження ясен і слизової ускладнюються запаленням, що протікає тривало. Встановлено, що у крові дитини практично відсутні імуноглобуліни усіх класів. Зниження функціональної активності якої клітинної популяції лежить в основі описаного синдрому? In the second year of his life, the boy often began to suffer from respiratory diseases, stomatitis, pustular skin lesions. Even minor damage to the gums and mucous membrane is complicated by inflammation that continues for a long time. It was established that that there are practically no immunoglobulins of all classes in the child's blood. A decrease in the functional activity of which cell population underlies the described syndrome?

В-лімфоцити B-lymphocytes

Макрофаги Macrophages

NK-лімфоцити NK-lymphocytes

Т-лімфоцити T-lymphocytes

Нейтрофіли Neutrophils

193 / 200
Секреція грудного молока у жінок обумовлена полімерними генами, причому кількість молока зростає із збільшенням числа домінантних алелів цих генів у генотипі жінки. Який генотип може мати породілля з відсутністю молока? Secretion of breast milk in women is determined by polymer genes, and the amount of milk increases with the increase in the number of dominant alleles of these genes in a woman's genotype. What genotype can a woman in labor have without milk?'

M1m1M2m2 M1m1M2m2

m1m1m2m2 m1m1m2m2

M1m1m2m2 M1m1m2m2

m1m1M2m2 m1m1M2m2

M1 M1m2m2 M1 M1m2m2

194 / 200
У хворого при томографічному обстеженні було виявлено пухлину відділу мозку з ушкодженням ядер XI та XII пар черепних нервів. Який це відділ мозку? The patient's tomographic examination revealed a brain tumor with damage to the nuclei of the XI and XII pairs of cranial nerves. What is the brain region?

Mesencephalon Mesencephalon

Myelencephalon Myelencephalon

Telencephalon Telencephalon

Metencephalon Metencephalon

Diencephalon Diencephalon

195 / 200
У хворого 69-ти років на шкірі в ділянці нижньої повіки з’явилося невелике бляшкоподібне утворення з наступним виразкуванням, яке було оперативно видалене. При мікроскопічному дослідженні утворення: в дермі шкіри комплекси з атипових епітеліальних клітин; периферії клітини розташовані перпендикулярно до базальної мембрани. Клітини темні, призматичної полігональної форми з гіперхромними ядрами з частими мітозами. Іноді зустрічаються утворення, подібні до волосяного фолікула. Яка гістологічна форма рака у хворого? In a 69-year-old patient, a small plaque-like formation appeared on the skin in the area of the lower eyelid, followed by ulceration, which was surgically removed. Upon microscopic examination of the formation: in dermis of the skin, complexes of atypical epithelial cells; the periphery of the cells are located perpendicular to the basement membrane. Cells are dark, prismatic polygonal in shape with hyperchromic nuclei with frequent mitoses. Sometimes there are formations similar to a hair follicle. What is the histological form of cancer in the patient?

Плоскоклітинний рак з ороговінням Squamous cell carcinoma with keratinization

Аденокарцинома Adenocarcinoma

Недиференційований рак Undifferentiated cancer

Плоскоклітинний рак без ороговіння Squamous cell carcinoma without keratinization

Базально-клітинний рак Basal cell carcinoma

196 / 200
До пологового відділення госпіталізували жінку зі слабкістю пологової діяльності. Який засіб необхідно використати для стимуляції скорочень матки? A woman was hospitalized in the maternity ward with weak labor activity. What means should be used to stimulate uterine contractions?

Іонадоліберин Ionadoliberin

Вазопресин Vasopressin

Кортиколіберин Corticoliberin

Соматостатин Somatostatin

Окситоцин Oxytocin

197 / 200
У стоматолога на прийомі дуже неспокійний пацієнт, який ніяк не може зручно влаштуватися в кріслі, норовить схопити доктора за руку, заглядає на маніпуляційний стіл, цікавиться в медсестри, чи стерильні інструменти. Який темперамент у цього пацієнта? The dentist has a very restless patient at the reception, who cannot sit comfortably in the chair, tries to grab the doctor's hand, looks at the manipulation table, asks the nurse whether sterile instruments. What temperament does this patient have?

Меланхолік Melancholic

Холерик Choleric

Сангвінік Sanguine

- -

Флегматик Phlegmatic

198 / 200
У хворого на слизовій оболонці ясен виразка овальної форми з припіднятими краями хрящоподібної щільності. Дно виразки м’ясисто-червоного забарвлення з нашаруваннями сірого кольору. При мікроскопічному дослідженні - проліферація ендотелію дрібних судин, периваскулярна лімфоплазмоцитарна інфільтрація. Про яке захворювання йдеться? The patient has an oval-shaped ulcer on the mucous membrane of the gums with raised edges of cartilaginous density. The bottom of the ulcer is fleshy-red in color with gray layers. Microscopic examination shows proliferation endothelium of small vessels, perivascular lymphoplasmacytic infiltration. What disease are we talking about?

Сифіліс Syphilis

Ерозивно-виразкова лейкоплакія Erosive-ulcerative leukoplakia

Травматична виразка Traumatic ulcer

Виразка-рак Ulcer-cancer

Виразково-некротичний гінгівіт Ulcerous-necrotic gingivitis

199 / 200
Визначте пульсовий і середньодинамічний артеріальний тиск (мм рт.ст.) у обстежуваного, якщо виміряний у нього артеріальний тиск становить 130/70 мм рт.ст.: Determine the pulse and average dynamic arterial pressure (mm Hg) of the subject, if the measured arterial pressure is 130/70 mm Hg:

60, 90 60, 90

60, 100 60, 100

50,90 50.90

60,80 60,80

50, 70 50, 70

200 / 200
У людини подразнення шкіри - свербіж, висипання, депігментація, збільшені лімфовузли. В оці знайдені філярії. Встановлений діагноз - онхоцеркоз. Які компоненти гнусу могли стати переносниками філярій p.Onchocerca? A person has skin irritation - itching, rash, depigmentation, enlarged lymph nodes. Filariae were found in the eye. The diagnosis was onchocerciasis. What components of disgust could become carriers of filariae p. Onchocerca?

Ґедзі Geji

Москіти Mosquitoes

Комарі Mosquitoes

Мокрець Mokrets

Мошки Midges